Section 7 Hematology

500

SECTION 7 Hematology

501

CHAPTER 25

502

Anemia

503 Case: A 41-year-old man with pleuritic chest pain A 41-year-old military veteran with a history of traumatic brain emergency department with lethargy. He has been fatigued ever since describes progressive shortness of breath and left-sided chest pain 108/74 mm Hg supine (which drops,to 94/63 mm Hg on standing). The injury complicated by ataxia and frequent falls presents to the sustaining a mechanical ground-level fall a few days ago. He also with inspiration. Heart rate is 122 beats per minute and blood pressure is skin and mucous membranes are pale; the patient’s hand is shown below the examiner’s hand in Figure 25-1A. There is a 2/6 early-

peaking crescendo-decrescendo systolic murmur best heard over the hemithorax with Decreased tactile fremitus and no audible breath base of the heart. There is dullness to percussion over the left sounds. Hemoglobin (Hb) is 6.2 mg/dL (there is no recent baseline for Corrected reticulocyte count is 21%. Total serum bilirubin, normal limits. Peripheral blood smear shows.many reticulocytes. comparison) with a mean corpuscular volume (MCV) of 97 fL. haptoglobin, and lactate dehydrogenase (LDH) levels are within Chest radiograph is shown in Figure 25-1B

FIGURE 25-1

What is the most likely cause of anemia in this patient?

What is anemia?

Are hemoglobin concentration, hematocrit, and red blood cell count always congruent in anemic patients? What is the role of the red blood cell in aerobic metabolism? Which formula describes the effects of anemia on tissue

Anemia is defined by a reduced absolute quantity of circulating red blood cells (RBCs) in the setting of normal plasma volume. It can be identified and measured with several laboratory tests, including hemoglobin concentration, hematocrit, or RBC count. In practice, hemoglobin concentration and hematocrit are most commonly used. Hematocrit is the percentage of packed blood containing intact RBCs after it has been spun in a centrifuge. The hematocrit is approximately 3 times the value of the hemoglobin concentration.1 In the setting of anemia, hemoglobin concentration, hematocrit, and RBC count usually decrease in parallel. In patients with profound microcytic anemia, such as thalassemia, the RBC count may be unexpectedly Increased, a clue to the diagnosis.2

Aerobic metabolism requires that oxygen and nutrients be exchanged for carbon dioxide and other waste products. RBCs carry oxygen from the lungs to metabolizing tissues, and carbon dioxide from the tissues back to the lungs where it is eliminated. Hemoglobin is the molecule found inside RBCs that allows for the transport of oxygen and carbon dioxide.3 The effect of anemia on tissue oxygenation is demonstrated in the 
formula that defines the oxygen content of arterial blood (Cao2): Cao2 =1.34 ×[Hb] ×Sao2 +(0.003 ×Pao2) In the above formula, 1.34 is the oxygen-binding capacity of hemoglobin (mL O2/g of Hb), [Hb] is the hemoglobin concentration

504

oxygenation?

What is normal hemoglobin concentration?

How is the severity of anemia defined?

How is red blood cell production regulated?

in blood (g/dL), Sao2 is the fraction of oxygenated hemoglobin in arterial blood, 0.003 is the solubility coefficient of O2 in blood (mL O2/100 mL blood/mm Hg Pao2), and Pao2 is the partial pressure of oxygen in arterial blood (mm Hg).3A 50% reduction in hemoglobin concentration (from 14 g/dL → 7 g/dL) results in a 50% reduction in Cao2, whereas a similar 50% reduction in Pao2 (from 90 mm Hg → 45 mm Hg) results in only a 20% decrease in Cao2. Normal hemoglobin concentration varies by sex, age, race, and environmental factors (eg, altitude). The lower limit of normal in young white men (ages 20-59 years) is 13.7 g/dL; in older white men (>60 years), it is 13.2 g/dL; in white women of all ages, it is 12.2 g/dL. These values are 0.5 to 1 g/dL lower in blacks. Pregnancy lowers the threshold by an additional 0.5 to 1 g/dL. Around 5% of normal individuals in these populations will have hemoglobin concentrations below these thresholds.4,5 Severity of anemia is variably defined (and dependent on factors such as sex, age, and race); however, the following thresholds provide a rule of thumb: Hemoglobin ≥9.5 g/dL is mild; hemoglobin 8 to 9.4 is moderate; hemoglobin <8 g/dL is severe.6 The average RBC has a life span of 120 days; production of new RBCs (erythropoiesis) replaces approximately 1% of circulating RBCs on a daily basis. Erythropoiesis normally occurs within the bone marrow and is stimulated by the hormone erythropoietin (EPO), which is produced and secreted primarily by the kidney. Impaired oxygen delivery to the kidney as a result of either anemia or hypoxemia (or rarely, Decreased blood flow from renal artery stenosis) provides the stimulus for EPO production (Figure 25-2).7,8

FIGURE 25-2 Impaired oxygen delivery to the kidney triggers EPO production, which increases the oxygen carrying

What conditions may lead to elevated baseline hemoglobin concentration? Which acute conditions can affect the laboratory evaluation of anemia? How common is anemia? What are some of the physiologic adaptations that occur as a result of anemia? What are the symptoms of anemia?

What are the physical findings of anemia? What are the red blood cell indices?

capacity of blood by stimulating erythropoiesis. The negative feedback loop is completed when the increase in oxygen carrying capacity improves oxygen delivery to the kidney, reducing EPO production. (Adapted with permission from McConnell TH, Hull:KL. Human Form, Human Function: Essentials of Anatomy and Physiology. Philadelphia, PA Lippincott Williams & Wilkins; 2011.)

Baseline hemoglobin concentration in an individual may be elevated as a compensatory response to chronic smoking, high altitude, or other medical conditions that lead to chronic hypoxemia (eg, chronic obstructive pulmonary disease). New laboratory values should be compared with recent baseline values if possible.

The RBC count, hemoglobin concentration, and hematocrit can be influenced by plasma volume. These laboratory values may be falsely elevated by hypovolemia and falsely lowered by hypervolemia. Acute and congruent changes involving all 3 blood cells of the complete blood count (white blood cells, RBCs, and platelets) can provide a clue to the presence of hemoconcentration or hemodilution. New laboratory values should be compared with recent baseline values if possible. Causes of hemodilution include intravenous fluid administration, heart failure, renal failure, and pregnancy. Anemia affects up to one-third of the world’s population and is often associated with significant morbidity and mortality. Compared with men, prevalence is higher in women, particularly during pregnancy.1,5 Physiologic adaptations to anemia begin after a period of days to weeks, and include Increased cardiac output (from Increased stoke volume and heart rate); Increased 2,3-biphosphoglycerate (2,3-BPG) production, which shifts the oxygen-hemoglobin dissociation curve to the right; Decreased systemic vascular resistance; and Increased coronary and cerebral blood flow.7,9 The clinical features of anemia depend on the severity, chronicity, and rate of onset. Mild anemia is frequently asymptomatic, but may be incidentally diagnosed on routine laboratory measurement. When gradual in onset, even severe anemia can remain subtle as a result of physiologic adaptation. Symptoms of anemia may include fatigue, loss of stamina, and dyspnea.7 Physical findings of anemia may include tachycardia, wide pulse pressure, forceful heartbeat, strong peripheral pulses, systolic flow murmur, and pallor of the mucous membranes and skin—particularly in locations where vessels are close to the surface (eg, palmar creases and nail beds).7 The RBC indices are part of a routine complete blood count (CBC) 
and include the MCV in femtoliters, mean cell hemoglobin 
(MCH) in picograms per cell, and the mean cell hemoglobin concentration (MCHC) in grams per

505

What are the 3 general categories of anemia based on mean corpuscular volume?

How is mean corpuscular volume determined?

deciliter.iCharacteristic changes to the MCV occur with certain causes of anemia, making it valuable in narrowing the hemoglobin synthesis.7 different al diagnosis. Low MCH/MCHC is referred to as hypochromia, which is consistent with impaired Anemia can be microcytic, normocytic, or macrocytic.

MCV can be calculated using data from the CBC or by reviewing the peripheral blood smear. An automated blood cell counter calculates it as follows:MCV (fL) =[hematocrit (percent) ×10]/[RBC count (106/µL)]Although the calculation is highly accurate, the peripheral blood smear is more sensitive to changes in MCV (Figure 25-3). Microcytosis is defined by an MCV <80 fL; macrocytosis is defined by an MCV >100 fL.10

FIGURE 25-3 Peripheral blood smears demonstrating changes in RBC size. Normal RBCs are approximately the same size as the nuclei of small lymphocytes (8 μm in diameter). The top panel shows numerous microcytic RBCs that are significantly smaller than the diameter of the lymphocyte nucleus (indicated by the lines and arrows). The microcytic cells are also hypochromic, with a large region of central pallor and just a thin peripheral rim that is hemoglobinized. The bottom panel demonstrates macrocytic RBCs (arrows). The one on the left is a polychromatophilic RBC (prematurely released reticulocyte). (From Weksler BB, Schechter GP, Ely SA. Wintrobe’s Atlas of Clinical Hematology. 2nd ed. Philadelphia, PA: Wolters Kluwer; 2018.)

506

Microcytic Anemia

What are the causes of microcytic anemia?

A previously healthy 28-year-old woman presents with fatigue and is found to have mild microcytic anemia with a serum ferritin of 7 ng/mL. A 28-year-old Greek man with microcytic anemia and codocytes (ie, target cells) on peripheral blood smear. Ring around the nucleus. Analysis of Ludwig van Beethoven’s hair and bones supports the theory that this condition may have been responsible for his death, possibly related to chronic consumption of adulterated wine.

What is the most powerful noninvasive test to evaluate for iron- deficiency anemia?

What is thalassemia?

Iron deficiency.

Thalassemia.

Sideroblastic anemia. Lead poisoning. Lead was often used illegally during Beethoven’s era to improve the taste of inexpensive wine.11,12

Iron-deficiency anemia is the most common cause of anemia worldwide. Causes include insufficient intake (eg, malnutrition), Decreased absorption (eg, celiac disease), chronic blood loss (eg, inflammatory bowel disease), and Increased demand (eg, pregnancy). Initial laboratory clues to the diagnosis include microcytosis and hypochromia. Serum ferritin is the single most powerful noninvasive test to evaluate for iron-deficiency anemia. Baseline ferritin values decline with age. Values may be Increased in the presence of chronic kidney disease, liver disease, or inflammatory conditions (eg, infection, rheumatoid arthritis, malignancy). However in any patient, a serum ferritin value <15 ng/mL confirms the diagnosis of iron deficiency, while a value of >100 ng/mL rules it out. Values between 15 and 100 require additional consideration, and other laboratory tests may be helpful, including the ratio of serum iron to total iron-binding capacity (ie, transferrin saturation).5,13,14 Thalassemia refers to a group of inherited conditions that result in Decreased or absent synthesis of 1 of the 2 polypeptide chains (α or β) that constitute the adult hemoglobin molecule (hemoglobin A, α2/β2). The inheritance pattern is autosomal recessive, and there are a variety of genotypes. Clinical severity ranges from asymptomatic disease (eg, α- thalassemia minor, β-thalassemia minor) to the presence of hepatosplenomegaly and chronic transfusion-dependent hemolytic anemia (eg, β-thalassemia major). Most patients with thalassemia have at least some degree of anemia, microcytosis, and hypochromia. Increased RBC count may be a clue to the diagnosis. In addition to hypochromic microcytosis, the peripheral blood smear may demonstrate poikilocytes (abnormal variation in RBC shape) such as target cells and teardrop-shaped RBCs (Figure 25-4), basophilic stippling, and nucleated RBCs (see Figure 27-5). Hemoglobin evaluation (separation and measurement of the hemoglobin fractions) or genetic testing can confirm the diagnosis.15

507

FIGURE 25-4 Peripheral blood smear from a patient with β-thalassemia minor demonstrating hypochromic microcytosis,

What is sideroblastic anemia?

What characteristic finding may be present on peripheral blood smear in patients with lead poisoning?

target cells (arrows), and teardrop-shaped RBCs (arrowhead). (From Weksler BB, Schechter GP, Ely SA. Wintrobe’s Atlas of Clinical Hematology. 2nd ed. Philadelphia, PA: Wolters Kluwer; 2018.)

Sideroblastic anemia refers to a group of inherited and acquired conditions characterized by impaired heme biosynthesis and the distinctive presence of ring sideroblasts (RBC precursors with excessive mitochondrial iron accumulation) in the bone marrow. Acquired sideroblastic anemias are significantly more common than the hereditary varieties; underlying etiologies include toxins (eg, alcohol), medication (eg, isoniazid), nutritional deficiencies (eg, copper deficiency), and myelodysplastic syndromes. Sideroblastic anemias tend to produce hemoglobin in the 4 to 10 g/dL range. The peripheral blood smear typically demonstrates hypochromic microcytosis, although normocytosis and macrocytosis are both possible, and basophilic stippling is occasionally seen.16 In addition to hypochromic microcytosis, the peripheral blood smear in patients with lead poisoning may reveal basophilic stippling: visible inclusions of ribosomes that appear as small dots in the periphery of RBCs (see Figure 41-5). Lead toxicity causes anemia because it inhibits heme biosynthesis and increases the fragility of RBCs, resulting in hemolysis.12

508

Normocytic Anemia

What laboratory test is helpful in subcategorizing the causes of normocytic anemia?

What are reticulocytes?

What does the reticulocyte count indicate about the function of the bone marrow?

The reticulocyte count of blood can determine whether normocytic anemia is hypoproliferative (low or normal reticulocyte count) or hyperproliferative (elevated reticulocyte count).

Reticulocytes are immature RBCs that are produced in bone marrow, released into the bloodstream, and mature into RBCs within 24 to 36 hours. Reticulocytes are larger than mature RBCs (see Figure 25-3). If the reticulocyte count is significantly elevated, it can tip the balance toward macrocytosis.7 The normal reticulocyte count is 1% to 2%, a reflection of normal daily RBC turnover. In the setting of anemia, an increase in reticulocyte count suggests that the bone marrow is responding appropriately by increasing production of RBCs; a reticulocyte count <2% (even if it is in the “normal” range) suggests that the bone marrow is not responding appropriately. To use the reticulocyte count as an index of RBC production, it must be corrected for anemia (ie, corrected reticulocyte count).7

509

Normocytic, Hypoproliferative Anemia

What are the causes of normocytic, hypoproliferative anemia?

A 56-year-old woman with diabetes and chronic osteomyelitis of the left great toe develops moderate, normocytic anemia over a period of several months. The bone marrow cannot produce red blood cells without a stimulus. Weight gain, constipation, dry skin, and depression. A group of conditions that usually cause pancytopenia.

What are the characteristics of anemia of inflammation?

What are the characteristics of anemia of chronic kidney disease?

What are the characteristics of anemia of hypothyroidism?

What is bone marrow failure?

Anemia of inflammation (ie, anemia of chronic disease).

Insufficient erythropoietin from chronic kidney disease (CKD).

Hypothyroidism.

Bone marrow failure.

Anemia can occur as a result of acute or chronic inflammation through a variety of mechanisms, including changes in iron homeostasis (eg, limited iron availability for erythropoiesis), impaired proliferation and differentiation of erythroid progenitor cells, Decreased production of and response to EPO, and shortened life span of RBCs. Anemia of inflammation is typically mild to moderate in severity, normochromic, and normocytic. Serum iron and transferrin saturation are both reduced. Serum ferritin is normal or Increased. Anemia of inflammation can sometimes present with mild microcytosis.6 Anemia of CKD mainly occurs as a result of EPO deficiency. It is typically normochromic and normocytic, with an incidence and severity proportional to the degree of underlying kidney disease. When hemoglobin targets are modest (eg, 10-11.5 g/dL), treatment with erythropoiesis-stimulating agents (eg, darbepoetin) is associated with improved symptoms from anemia, improved quality of life, and Decreased need for blood transfusions. However, higher hemoglobin targets (>13 g/dL) are associated with adverse outcomes, including stroke and hypertension.17 Anemia occurs in almost one-half of patients with overt or subclinical hypothyroidism and is frequently the presenting manifestation. It is typically normochromic and normocytic, with a severity proportional to the degree of hypothyroidism. Mechanisms include marrow hypoproliferation from thyroid hormone deficiency, and Decreased EPO production as a result of reduced oxygen requirements associated with the hypothyroid state. Iron deficiency is associated with hypothyroidism as a result of Decreased Gastrointestinal iron absorption and menorrhagia in women. Pernicious anemia is 20 times more common in patients with hypothyroidism and can result in vitamin B12 deficiency. These associated conditions can give rise to microcytic and macrocytic anemias in patients with hypothyroidism.18,19 Bone marrow failure refers to a group of inherited and acquired conditions characterized by intrinsic dysfunction of hematopoiesis, usually resulting in pancytopenia. Causes of acquired bone marrow failure include aplastic anemia and infiltrative conditions (eg, leukemia, myelofibrosis) (see chapter 27, Pancytopenia). Pure red cell aplasia is a type of inherited or acquired bone marrow failure that presents with isolated anemia. Associated conditions include infection (eg, parvovirus B19), hematologic malignancies (eg, large granular lymphocyte leukemia), thymoma, medications (eg, phenytoin), and collagen vascular diseases (eg, systemic lupus erythematosus). The anemia is typically moderate to severe, normochromic, and normocytic, with a profound reticulocytopenia. Bone marrow is typically normocellular or hypocellular with a selective decrease or absence of erythroblasts.20

510

511

Normocytic, Hyperproliferative Anemia

What are the causes of normocytic, hyperproliferative anemia?

A 48-year-old man Gastrointestinal bleeding caused by peptic ulcer disease. who has been taking ibuprofen regularly for the past month for low back pain presents with acute-onset hematemesis, black stools, light-headedness, and hypotension. A 29-year-old man Hemolytic anemia from glucose-6-phosphate dehydrogenase (G6PD) deficiency. presents with jaundice, dark urine, and normocytic anemia after starting trimethoprim-sulfamethoxazole for a soft tissue infection. May be palpable on Splenomegaly. physical examination.

What are the characteristics of anemia caused by acute hemorrhage?

What are the laboratory features of hemolytic anemia?

How does splenomegaly cause anemia?

The effects of hypovolemia, including hypotension and Decreased organ perfusion, dominate the clinical picture of acute blood loss anemia. Healthy young adult men in the third decade of life have approximately 6 L of total blood volume, which decreases to around 5 L by the seventh decade of life. Acute blood loss of up to 30% of total blood volume can manifest tachycardia and tachypnea, however blood pressure is typically normal or only mildly Decreased. Acute blood loss >30% of total blood volume causes orthostatic hypotension that may progress to hypotensive shock, and is frequently associated with dyspnea, diaphoresis, cold and clammy skin, Decreased urine output, and delayed capillary refill. Acute blood loss may not initially result in Decreased hemoglobin concentration. Chronic blood loss anemia may gradually deplete iron stores and result in microcytic iron-deficiency anemia.7,21 Hemolytic anemia is typically characterized by hyperproliferative anemia associated with evidence of RBC destruction, such as Increased serum indirect bilirubin, Increased serum aspartate aminotransferase, Increased serum LDH, Increased free serum hemoglobin, Decreased serum haptoglobin, hemoglobinuria, and Increased urine and stool urobilinogen. The pattern of laboratory abnormalities depends on whether hemolysis is intravascular or extravascular (see chapter 26, Hemolytic Anemia). The peripheral blood smear may demonstrate evidence of hemolysis (eg, reticulocytosis, polychromasia, nucleated RBCs) and other features suggestive of certain causes of hemolytic anemia (eg, schistocytes, spherocytes, bite cells). Hemolysis may be associated with macrocytic anemia because reticulocytes are larger than mature RBCs.7,10 Splenomegaly and associated hypersplenism cause Decreased hemoglobin concentration via 3 main mechanisms. First, there is an Increased rate of RBC destruction (ie, hemolysis) within the enlarged spleen. Second, there is sequestration of RBCs; massively enlarged spleens are capable of sequestering as much as one-third of the total RBC mass. Third, splenomegaly causes an increase in plasma volume, resulting in a dilutional effect that decreases hemoglobin concentration. Splenomegaly is considered massive when it reaches the iliac crest, crosses the abdominal midline, or weighs more than 1500 g. The most common causes of massive splenomegaly include hematologic disorders (eg, chronic myeloid leukemia, primary myelofibrosis, polycythemia vera, indolent lymphoma, hairy cell leukemia, β-thalassemia major), infections (eg, visceral leishmaniasis [kala-azar], malaria), and infiltrative diseases (eg, Gaucher disease).22,23

512

Macrocytic Anemia

What are the 2 subcategories of macrocytic anemia?

What is megaloblastic anemia?

What are the features of megaloblastic anemia on peripheral blood smear?

Macrocytic anemia can be megaloblastic or nonmegaloblastic.

Megaloblastic anemia occurs as a result of impaired deoxyribonucleic acid (DNA) synthesis that interferes with RBC proliferation and maturation. It is associated with characteristic features on peripheral blood smear and bone marrow aspirate. The impaired DNA synthesis of megaloblastic anemia affects all developing blood cells, often resulting in pancytopenia. Intramedullary lysis of erythroid progenitors can result in manifestations such as jaundice and elevated serum LDH levels.10 The peripheral blood smear of megaloblastic anemia is characterized by the presence of large (MCV often >115 fL) and oval-shaped RBCs (macro-ovalocytes), hypersegmented neutrophils, anisocytosis (abnormal variation in RBC size), and poikilocytosis (eg, teardrop-shaped RBCs) (Figure 25-5). With the exception of myelodysplasia, nonmegaloblastic causes of macrocytosis generally result in round macrocytes.10

FIGURE 25-5 Peripheral blood smear from a patient with megaloblastic anemia due to vitamin B12 deficiency demonstrating

What are the features of megaloblastic anemia on bone marrow evaluation?

many macro-ovalocytes (arrows) with anisocytosis, hypersegmented neutrophils (stars), and teardrop-shaped RBCs (arrowhead). Neutrophils are considered hypersegmented when more than 5% have 5 or more lobes, or when there are occasional neutrophils with 6 or more lobes. (From Weksler BB, Schechter GP, Ely SA. Wintrobe’s Atlas of Clinical Hematology. 2nd ed. Philadelphia, PA: Wolters Kluwer; 2018.)

Megaloblastic anemia is associated with marrow hypercellularity and evidence of abnormal proliferation and maturation of multiple myeloid cell lines, including large, oval-shaped erythroblasts that contain a characteristic finely stippled, lacy nuclear chromatin pattern surrounded by normal-appearing cytoplasm (ie, nuclear-cytoplasmic dissociation). Such progenitor cells are referred to as “megaloblasts.” Nonmegaloblastic conditions such as myelodysplasia and leukemia may be associated with “megaloblastic-like” abnormalities of the bone marrow, but can usually be distinguished with careful examination.10,24

513

Megaloblastic Anemia

What are the causes of megaloblastic anemia?

A 68-year-old woman with Hashimoto’s thyroiditis and vitiligo develops macrocytic anemia and ataxia. Green leafy vegetables are an important part of a healthy diet. A patient with Wilson’s disease who is taking more than the recommended doses of penicillamine and zinc acetate develops megaloblastic anemia and ataxia, and is found to have a positive Romberg test with brisk knee reflexes and absent ankle reflexes. A 23-year-old woman presents with megaloblastic anemia and admits to regularly tampering with whipped cream canisters to get high.

Vitamin B12 deficiency caused by pernicious anemia.

Folate deficiency. Copper deficiency related to excess penicillamine (a copper-chelating agent) and zinc.

Nitrous oxide inhalation (called “whippets”).

What are the nonhematologic In addition to megaloblastic anemia, vitamin B12 deficiency often causes myeloneuropathy, the features of vitamin B12 combination of spinal cord disease and peripheral neuropathy. The earliest clinical manifestations deficiency? include ataxia and paresthesias in the extremities; other features include sensory loss, lower limb weakness, spasticity, hyperreflexia and/or hyporeflexia (eg, brisk patellar reflex with diminished Achilles reflex), cognitive impairment, and vision loss.25 What serum laboratory tests In the context of a compatible clinical condition, plasma homocysteine and methylmalonic acid can be can be helpful in diagnosing useful in the diagnosis of megaloblastic anemia. In patients with vitamin B12 deficiency, both and differentiating metabolites are typically elevated, whereas in patients with folate deficiency, only homocysteine is megaloblastic anemia related elevated.10 to vitamin B12 and folate deficiency? How common is folate Folate deficiency is rare in countries that fortify food with folic acid. However, some patients may have deficiency in the industrialized predisposing conditions that lead to folate deficiency, including poor nutritional intake (eg, eating world? disorders), Gastrointestinal disease (eg, bariatric surgery), chronic alcohol use, chronic hemolysis (eg, sickle cell anemia), and conditions with high cellular turnover (eg, exfoliative dermatitis). In such patients it may be necessary to provide long-term prophylactic folic acid supplementation.10 What are the risk factors for Conditions that predispose patients to developing copper deficiency include gastrectomy, use of copper- copper deficiency? chelating agents (eg, penicillamine), excessive zinc intake (due to zinc’s interference with copper absorption in the small intestine), Gastrointestinal disease (eg, inflammatory bowel disease), and chronic enteral or parenteral nutrition with insufficient copper supplementation.25 What are the general Drugs and toxins cause megaloblastic anemia by impairing either the cellular availability or use of mechanisms of megaloblastic vitamin B12 or folic acid. Nitrous oxide blocks the conversion of vitamin B12 from the reduced to the anemia related to drugs or oxidized form, leading to impaired DNA synthesis. The list of drugs and toxins that can cause or toxins? contribute to megaloblastic anemia is extensive; some of the common ones include alcohol, immunomodulators (eg, azathioprine, mycophenolate), antineoplastic agents (eg, methotrexate, hydroxyurea), antimicrobial agents (eg, tetracyclines, penicillins, trimethoprim), antiseizure agents (eg, phenytoin), allopurinol, colchicine, metformin, and proton-pump inhibitors. Medications that affect DNA synthesis (eg, hydroxyurea, azathioprine) can cause macrocytosis with or without megaloblastic changes.24,26

514

Nonmegaloblastic Macrocytic Anemia

What are the causes of nonmegaloblastic macrocytic anemia?

Patients who consume this toxin in excess are also at risk for folate deficiency and liver disease. A 48-year-old woman with a history of chronic hepatitis C infection presents with jaundice, multiple spider angiomas on the chest, and mild, macrocytic anemia. A hematopoietic disease of the elderly. A high percentage of immature RBCs.

How severe is the macrocytosis of alcoholism?

What are the characteristics of the macrocytosis associated with liver disease?

Alcohol.

Liver disease.

Myelodysplasia. Reticulocytosis.

Macrocytosis without anemia can occur with regular consumption of 30 to 40 g of alcohol (half a bottle of wine) daily, particularly in women. Anemia eventually develops, especially in patients with higher alcohol consumption (eg, >80 g/d). The mechanism of alcohol-related macrocytosis is unknown. The macrocytes are round, and the degree of severity is typically modest; the vast majority of cases are associated with MCV <110 fL when concomitant liver disease or megaloblastic anemia is absent.26,27 Patients with macrocytosis related to liver disease virtually always have stigmata of advanced liver disease on examination (eg, jaundice, spider angiomas) and abnormal liver function tests (eg, aminotransferase elevation). Concomitant thrombocytopenia and/or leukopenia is common. The round macrocytosis of nonalcoholic liver disease is typically modest with an average MCV of 105 fL; the average MCV of alcoholic liver disease is higher at 108 fL. The peripheral blood smear may also demonstrate target cells or acanthocytes (including spur cells) (Figure 25-6).27

FIGURE 25-6 Peripheral blood smear from a patient with alcoholic liver disease demonstrating round

Why is myelodysplasia sometimes confused for megaloblastic anemia? What degree of reticulocytosis is associated with macrocytosis? What are other causes of nonmegaloblastic macrocytosis?

macrocytosis and target cells. (From Pereira I, George TI, Arber DA. Atlas of Peripheral Blood: The Primary Diagnostic Tool. Philadelphia, PA: Wolters Kluwer Health; 2012.)

Both myelodysplasia and B12 deficiency are diseases of the elderly and often present with pancytopenia. Myelodysplasia is associated with megaloblastic-like changes of the bone marrow that can be confused for megaloblastic anemia. Careful examination is needed to make the distinction.10 The vast majority of cases of macrocytosis caused by reticulocytosis feature a reticulocyte count ≥10%. Most cases are related to hemolytic anemia (eg, sickle cell anemia).27 Other causes of nonmegaloblastic macrocytosis include pregnancy, hypothyroidism, aplastic anemia, pure red cell aplasia, acute leukemia, and multiple myeloma.10,26

515

516 Case Summary A 41-year-old man presents with fatigue and dyspnea following a an elevated reticulocyte count, and complete opacification of the left mechanical fall and is found to have severe normocytic anemia with hemithorax on chest imaging.

What is the most likely cause of anemia in this patient? Hemothorax.

517

Bonus Questions

Whatlis the The heart murmur in this case is likely a physiologic flow murmur related to the high output state of anemia. Other physical findings of anemia include tachycardia, cause of the heart murmu Figure 25-1A most ikely wide pulse pressure, forceful heartbeat, strong peripheral pulses, and pallor of the mucous membranes and skin (see ). 7 in this case? r What physical Physical findings of a pleural effusionfinclude Decreased inspiratory expansion of the chest wall on the affected side, dullness to percussion, Decreased tactile fremitus, presence of a suggest the findings and reduced breath sounds over the e fusion. Lung consolidation may also present with dullness to percussion, however tactile fremitus is usually Increased. pleural effusion? What are the In addition to pleural effusion, complete opacification of the hemithorax can occur as a result of lobar atelectasis from mucus plugging or endobronchial tumor. The completeax n? that area (ie,iatelectasis); tracheal deviation away from the opacified hemithorax suggests a volume-occupying process (eg, pleural effusion). In this case, there is causes of direction of the trachea on the chest radiograph can help distinguish these conditions. Tracheal deviation toward the opacified hemithorax suggests volume loss over opacificatio hemithor tracheal dev ation away from the opacified hemithorax (see Figure 25-1B), consistent with pleural effusion. What is the Given the associated severe anemia, the effusion in this case is most likely a hemothorax. An intercostal artery may have been disrupted by a rib fracture sustained most likely during the ground-level fall. Hemothorax is confirmed when the pleural fluid hematocrit level is greater than half that of blood. An estimate of the hematocrit of the cause of the fluid can be obtained by dividing the RBC count by 100,000 (eg, RBC count of 1,000,000 =hematocrit of 10). 28 pleural effusion in this case? What are the Hemothorax is most often the result of chest trauma, including iatrogenesis (eg, cardiopulmonary surgery). Spontaneous hemothorax is less common but can occur in causes of the context of anticoagulation therapy, tumors (eg, schwannoma, pleural metastases), rupture of pleural adhesions, pulmonary infarction, rupture of aneurysmatic hemothorax? thoracic arteries, rupture of pulmonary vascular malformations (eg, in patients with hereditary hemorrhagic telangiectasia), and thoracic endometriosis. Retention of blood in the pleural space can lead to chronic fibrothorax, trapped lung, impaired lung function, and infection. How should th In most cases, hemothorax should be treated with insertion of a large bore (≥28 French) chest tube to drain the blood and al ow for re-expansion of the lung. A chest hemothorax radiograph should be repeated after thoracostomy to identify the position of the chest tube, identify any intrathoracic pathology, and confirm that the blood has been 29 this case be ine fully drained. Intrapleural fibrinolytic therapy may be necessary to break down residual blood clots and pleural adhesions.lIn order to maintain hemodynamic stability, managed? a surgical approach may be necessary in patients with high rates of active bleeding that require repeated blood transfusions. 29

518 Key Points

Anemia is defined by a reduced absolute quantity of circulating Normal hemoglobin concentration varies by sex, age, race, and The clinical features of anemia depend on severity, chronicity, and Symptoms of anemia include fatigue, loss of stamina, and RBCs. other environmental factors. rate of onset. dyspnea. Physical findings of anemia include tachycardia, wide pulse

pressure, forceful heartbeat, strong peripheral pulses, systolic flow Anemia can be microcytic, normocytic, or macrocytic. hypoproliferative (reticulocyte count Decreased or inappropriately murmur, and pallor of the mucous membranes and skin. The reticulocyte count determines whether normocytic anemia is normal), or hyperproliferative (reticulocyte count Increased). Macrocytic anemia can be megaloblastic or nonmegaloblastic.

Anemia of inflammation is usually associated with normocytic Reticulocytosis can be associated with either normocytic or anemia but is sometimes associated with mild microcytosis. macrocytic anemia.

519

References 1. Sankaran VG, Weiss MJ. Anemia: progress in molecular mechanisms and therapies. Nat Med. 2015;21(3):221-230.

  1. Schriever HG. Red cell indices in thalassemia minor. Ann Clin Lab Sci. 1974;4(5):339- 342.

  2. Marino PL. The ICU Book. 3rd ed. Philadelphia, PA: Lippincott Williams & Wilkins; 2007.

  3. Beutler E, Waalen J. The definition of anemia: what is the lower limit of normal of the blood hemoglobin concentration? Blood. 2006;107(5):1747-1750.

  4. Lopez A, Cacoub P, Macdougall IC, Peyrin-Biroulet L. Iron deficiency anaemia. Lancet. 2016;387(10021):907-916.

  5. Weiss G, Goodnough LT. Anemia of chronic disease. N Engl J Med. 2005;352(10):1011- 1023.

  6. Longo DL, Fauci AS, Kasper DL, Hauser SL, Jameson JL, Loscalzo J, eds. Harrison’s Principles of Internal Medicine. 18th ed. New York, NY: McGraw-Hill; 2012.

  7. Shemin D, Rittenberg D. The life span of the human red blood cell. J Biol Chem. 1946;166(2):627-636.

  8. Hebert PC, Van der Linden P, Biro G, Hu LQ. Physiologic aspects of anemia. Crit Care Clin. 2004;20(2):187-212.

  9. Aslinia F, Mazza JJ, Yale SH. Megaloblastic anemia and other causes of macrocytosis. Clin Med Res. 2006;4(3):236-241.

  10. Stevens MH, Jacobsen T, Crofts AK. Lead and the deafness of Ludwig van Beethoven. Laryngoscope. 2013;123(11):2854-2858.

  11. Wani AL, Ara A, Usmani JA. Lead toxicity: a review. Interdiscip Toxicol. 2015;8(2):55- 64.

  12. Camaschella C. Iron-deficiency anemia. N Engl J Med. 2015;372(19):1832-1843. 14diagnosis of,iron-deficiencylanemia: an overview. J Gen Intern Med. 1992;7(2):145-153. 15. Peters M, Heijboer H, Smiers F, Giordano PC. Diagnosis and management of . Guyatt GH Oxman AD, A i M, Willan A, McIlroy W, Patterson C. Laboratory thalassaemia. BMJ. 2012;344:e228.

  13. Alcindor T, Bridges KR. Sideroblastic anaemias. Br J Haematol. 2002;116(4):733-743.

  14. Kalantar-Zadeh K, Aronoff GR. Hemoglobin.variability in anemia of chronic kidney 18. Antonijevic N, Nesovic M, Trbojevic B, Milosevic R. Anemia in hypothyroidism. Med disease. J Am Soc Nephrol. 2009;20(3):479-487 Pregl. 1999;52(3-5):136-140.

  15. Erdogan M, Kosenli A, Ganidagli S, Kulaksizoglu M. Characteristics of anemia in subclinical and overt hypothyroid patients. Endocr J. 2012;59(3):213-220.

  16. Leguit(RJ, van den Tweel JG. The pathology of bone marrow failure. Histopathology. 21. Davy KP, Seals DR. Total blood volume in healthy young and older men. J Appl 2010;57 5):655-670. Physiol (1985). 1994;76(5):2059-2062. . Hess CE, Ayers CR, Sandusky WR, Carpenter MA, Wetze RA, Mohler DN.

22Mechanism of dilutional anemia in massive splenomegaly.lBlood. 1976;47(4):629-644. 23. Paz YMHL, Gonzalez-Estrada A, Alraies MC. Massive splenomegaly. BMJ Case Rep. 2013;2013.

520

  1. Hesdorffer CS, Longo DL. Drug-induced megaloblastic anemia. N Engl J Med. 2015;373(17):1649-1658.

  2. Garg RK, Malhotra.HS, Kumar N. Approach to a case of myeloneuropathy. Ann 26. Hoffbrand V, Provan D. ABC of clinical haematology. Macrocytic anaemias. BMJ. Indian Acad Neurol 2016;19(2):183-187. 1997;314(7078):430-433. . Savage DG, Ogundipe A, Allen RH, Stabler SP, Lindenbaum J. Etiology a

27diagnostic evaluation of macrocytosis. Am J Med Sci. 2000;319(6):343-352. nd 28. Light RW. Pleural Diseases. 5th ed. Philadelphia, PA: Lippincott Williams & Wilkins; 2007.

  1. Boersma WG, Stigt JA, Smit HJ. Treatment of haemothorax. Respir Med. 2010;104(11):1583-1587.

521

CHAPTER 26

522

Hemolytic Anemia

523 Case: A 31-year-old woman with dark urine A previously healthy 31-year-old woman presents to the emergency colored urine. The symptoms developed during a trip to Denver for an rafting on the Snake River in Oregon, where she was bitten by a spider fresh fava beans a day before the symptoms began. She has not scHeart rate is 105 beats per minute. There is generalized jaundice and department with several days of fatigue, dyspnea, and dark tea- Ultimate Frisbee tournament. A few days before the trip, she went but reports no other significant exposures. She consumed a bowl of recently taken any medications or supplements. leral icterus. There is conjunctival pallor. No organomegaly is appreciated. Hemoglobin is 6.9 g/dL, mean corpuscular volume (MCV) is 98 fL, and total bilirubin is 7.5 mg dL with an indirect fraction of 7 mg/dL. The corrected reticulocyte count is 13%, lactate dehydrogenase (LDH

is 687 U/L, and haptoglobin/is not detectable. The direct antiglobulin ) 26-1. (Coombs) test is negative. Peripheral blood smear is shown in Figure

FIGURE 26-1 (Courtesy of Michael J. Cascio, MD.)

What is the most likely cause of hemolytic anemia in this patient?

What is hemolytic anemia? What is the average life span of a red blood cell? Where does hemolysis occur within the body?

What are the symptoms of hemolytic anemia? What are the mechanisms of discolored urine in patients

Hemolysis describes premature destruction of red blood cells (RBCs), leading to a decrease in the average life span of the cells in circulation. Hemolytic anemia occurs when hemolysis outpaces bone marrow production. The average RBC has a life span of approximately 120 days; production of new RBCs (erythropoiesis) replaces 1% of circulating RBCs on a daily basis. Erythropoiesis normally occurs within the bone marrow and is stimulated by the hormone erythropoietin (EPO), which is produced and secreted primarily by the kidney (see Figure 25-2).1–3

Hemolysis can be intravascular, extravascular, or both. Intravascular hemolysis occurs when RBC damage is severe enough to cause immediate lysis of cells within the intravascular space, which releases free hemoglobin into the bloodstream. Less severe RBC damage prompts the reticuloendothelial macrophages of the spleen and liver to remove the affected cells, generating unconjugated bilirubin from the breakdown of hemoglobin. Disorders of ineffective hematopoiesis, such as megaloblastic anemia, can lead to premature intramedullary lysis of erythroid progenitors, mimicking hemolytic anemia.4 The clinical features of hemolytic anemia depend on severity, chronicity, rate of onset, and location of hemolysis (intravascular or extravascular). Symptoms may include fatigue, loss of stamina, and dyspnea. Dark, “cola-colored” urine can indicate intravascular or severe extravascular hemolysis, red-colored urine can indicate rapid intravascular hemolysis, and flank pain can be present in abrupt-onset hemolysis.3,4 Intravascular and severe extravascular hemolysis results in the release of free hemoglobin into the bloodstream. It is oxidized to the darkly pigmented methemoglobin, which is filtered into urine, turning it a dark cola color. In cases of rapid intravascular hemolysis, fresh nonoxidized hemoglobin, which is bright red in color, is filtered into urine, giving it a reddish appearance. A common misconception is that bilirubin is the cause of the dark urine of hemolytic anemia (the unconjugated bilirubin generated by extravascular hemolysis is water-insoluble and is not filtered into urine).

524

with

hemolytic anemia? What are the physical findings of hemolytic anemia? What are the laboratory features of hemolytic anemia?

What are the clinical sequelae of chronic hemolysis? What are the 2 general ways in which patients develop hemolytic anemia?

What historical clues can help differentiate inherited and acquired causes of hemolytic anemia?

Physical findings of hemolytic anemia may include fever, tachycardia, wide pulse pressure, forceful heartbeat, strong peripheral pulses, systolic flow murmur, pallor of the mucous membranes and skin (particularly in the palmar creases and nail beds where vessels are close to the skin), and jaundice and splenomegaly (in extravascular hemolysis).3,4

Hemolytic anemia is typically a normocytic or macrocytic hyperproliferative anemia associated with evidence of RBC destruction such as Increased excretion of urobilinogen in urine and stool, Increased serum aspartate aminotransferase, Increased serum indirect bilirubin (particularly in extravascular hemolysis), as well as Increased serum LDH and Decreased serum haptoglobin (particularly in intravascular hemolysis). When intravascular hemolysis is predominant or extravascular hemolysis is severe, there is Increased free serum hemoglobin and hemoglobinuria (giving rise to cola-colored or red-colored urine). In hemolysis that is predominantly intravascular, serum bilirubin may be normal because only extravascular hemolysis involves the breakdown of hemoglobin into unconjugated bilirubin within reticuloendothelial macrophages. The peripheral blood smear may demonstrate evidence of hemolysis (eg, reticulocytosis, polychromasia, nucleated RBCs) and other features suggestive of certain causes of hemolytic anemia (eg, schistocytes, spherocytes, bite cells).3,5 Chronic hemolysis can lead to the development of folate deficiency, cholelithiasis with pigmented gallstones, splenomegaly, leg ulcers, and pulmonary hypertension.4

Hemolytic anemia can be inherited or acquired.

Patients with inherited hemolytic anemia are more likely to present at a younger age and may have a positive family history. However, a family history is not always present when the inheritance pattern is recessive or when the patient has a de novo mutation. Additionally, children frequently acquire hemolytic anemia (eg, hemolytic uremic syndrome [HUS]), and some inherited conditions do not present until adulthood.3

525

Inherited Hemolytic Anemia

What are the subcategories of inherited hemolytic anemia based on the structural components of red blood cells?

Inherited hemolytic anemia can be caused by defects that involve hemoglobin, intracellular enzymes, or the membrane-cytoskeleton complex.

526

Inherited Hemolytic Anemia Related to Hemoglobin Defects

What is hemoglobin?

What is hemoglobinopathy?

Hemoglobin is a molecule found in high concentrations inside RBCs and is responsible for carrying oxygen from the lungs to the rest of the tissues in the body. Hemoglobin A, which is composed of 2 α and 2 β subunits, is the main form of hemoglobin in normal adults. Each subunit of hemoglobin contains a heme moiety made up of a porphyrin ring with an iron atom at its center, which allows for the binding of oxygen.3 The hemoglobinopathies are a group of disorders characterized by defects in hemoglobin structure, function, or production. Solubility, reversible oxygen binding, and other important properties of hemoglobin may be deranged in hemoglobinopathies, leading to damaged RBCs and hemolysis.3

What are the inherited hemoglobinopathies?

527

528

529

Inherited Hemolytic Anemia Related to Intracellular Enzyme Defects

What are the primary roles of the red blood cell enzymes?

The 2 main functions of the RBC enzymes are the production of energy and the prevention of oxidative damage to hemoglobin and other proteins. Both processes are important for the maintenance of RBC organization, shape, structure, and survival.3,4

What are the inherited red blood cell enzyme defects?

A 33-year-old Syrian man develops jaundice and hemolytic anemia after eating warmed fūl (fava beans) mixed with olive oil, lemon juice, and garlic. This condition involves the enzyme that catalyzes the final step in glycolysis.

What peripheral smear findings are associated with G6PD deficiency during an episode of acute hemolysis?

Glucose-6-phosphate dehydrogenase (G6PD) deficiency.

Pyruvate kinase deficiency.

Peripheral smear findings associated with the hemolysis of G6PD deficiency include anisocytosis (abnormal variation in RBC size), polychromasia, irregularly contracted cells (which are hyperchromic-like spherocytes but can be distinguished by an irregular outline), Heinz bodies, and characteristic poikilocytes, including blister cells or “hemighosts” (Figure 26-3) and bite cells (cells that appear as if a semicircular bite has been taken from them, which occurs when the spleen removes Heinz bodies).

FIGURE 26-3 Peripheral blood smear from a patient with acute hemolysis related to G6PD deficiency,

What are the clinical features of pyruvate kinase deficiency?

demonstrating blister cells (arrows) and polychromatic RBCs (arrowheads). Blister cells appear as if hemoglobin has been pushed to one side of the cell and are usually only present when hemolysis is brisk.1 (Courtesy of Michael J. Cascio, MD.)

Pyruvate kinase deficiency is an uncommon disorder but is the most frequent defect of the glycolytic pathway. The degree of hemolysis between patients is highly variable, including some who are virtually asymptomatic and others who require regular blood transfusions. Hemolysis is mostly extravascular. Peripheral smear findings are nonspecific (eg, polychromasia, reticulocytosis). Splenectomy may be beneficial in patients with severe disease.3,4

530

Inherited Hemolytic Anemia Related to Cell Membrane Defects

What are the main functions of the red blood cell membrane?

The RBC membrane is designed to allow for a high degree of deformability, which is necessary for the cell to squeeze through microscopic structures; it is also responsible for maintaining cellular water balance. Defects in the RBC membrane may lead to the loss of cellular elasticity or dehydration or overhydration, shortening the life span of the cell.

What are the inherited Cell Membrane Defects?

The peripheral smear findings of this condition could also be consistent with autoimmune hemolytic anemia. Also known as ovalocytosis, this disease confers some resistance against malaria. Two conditions of abnormal red blood cell hydration.

What are the clinical features of hereditary spherocytosis?

What are the clinical features of hereditary elliptocytosis?

Hereditary spherocytosis.

Hereditary elliptocytosis.

Hereditary stomatocytosis and hereditary xerocytosis.

Hereditary spherocytosis is associated with a wide spectrum of disease, from severe hemolytic anemia starting in infancy to largely asymptomatic cases that present in adulthood. Presenting manifestations in adults may include jaundice, splenomegaly, or gallstones. Hereditary spherocytosis is one of the few conditions associated with an Increased mean corpuscular hemoglobin concentration (MCHC), which may be a clue to the diagnosis. Spherocytes are the characteristic finding on peripheral blood smear; these cells are smaller than normal RBCs and lack central pallor. Osmotic fragility testing may be helpful. The condition is definitively diagnosed with molecular testing.3 Hereditary elliptocytosis is clinically similar to hereditary spherocytosis with a wide range of disease severity. Elliptocytes are the characteristic finding on peripheral blood smear; these cells are elliptic or oval in shape (smaller than the oval macrocytes of megaloblastic anemia and myelodysplasia) (Figure 26-4).3

FIGURE 26-4 Peripheral blood smear demonstrating elliptocytes (arrows). Elliptocytes are elongated with rounded edges (as opposed to the sharp edges of sickle cells). (From Weksler BB, Schechter GP, Ely SA. Wintrobe’s Atlas of Clinical Hematology. 2nd ed. Philadelphia, PA: Wolters Kluwer; 2018.)

What are the Hereditary stomatocytosis and hereditary xerocytosis are both autosomal dominant conditions of abnormal RBC water

531

differences between hereditary stomatocytosis and hereditary xerocytosis?

content, thought to be driven by cation transport disturbances. Hereditary stomatocytosis describes RBC overhydration, which results in Decreased MCHC and the presence of stomatocytes (RBCs with slit-like central pallor) on peripheral blood smear. Hereditary xerocytosis describes RBC dehydration, which results in Increased MCHC and the presence of xerocytes (dense RBCs with marked hyperchromia) on peripheral blood smear. Osmotic fragility is Increased in hereditary stomatocytosis but Decreased in hereditary xerocytosis. Clinical severity is variable for both disorders. Splenectomy should be avoided in both conditions, as it increases the risk of thromboembolic complications. 3,4

532

Acquired Hemolytic Anemia

What are the 4 mechanisms of acquired hemolytic anemia?

Acquired hemolytic anemia can be immunologic, toxic, traumatic, or infectious.

533

Kluwer 2018.)

Immunologic Causes of Acquired Hemolytic Anemia

What are the Immunologic Causes of acquired hemolytic anemia?

Spherocytosis and a positive direct Coombs test. History of multiple blood transfusions. A 44-year-old woman who is hospitalized for pyelonephritis develops jaundice and anemia after starting ceftriaxone. A condition in which the largest lymphatic organ in the body becomes more active. Examples include acanthocytes (including spur cells) and echinocytes (burr cells). A 35-year-old man with several months of fatigue and episodes of red-colored urine that is most pronounced in the morning presents with acute abdominal pain and is found to have anemia, undetectable haptoglobin, and mesenteric vein thrombosis.

Autoimmune hemolytic anemia (AIHA). Alloimmune hemolytic anemia (eg, delayed hemolytic transfusion reaction). Drug-induced immune hemolytic anemia.

Hypersplenism.

Acquired erythrocyte membrane defects.

Paroxysmal nocturnal hemoglobinuria (PNH).

What are the clinical features of autoimmune hemolytic anemia?

AIHA is the most common acquired hemolytic anemia in countries where malaria is not endemic. It occurs when antibodies react with antigens on the surface of RBCs. It can be primary or secondary (eg, systemic lupus erythematosus, medication). Warm AIHA is associated with IgG antibodies, which react with RBC surface antigens at body temperature, causing extravascular hemolysis. Cold AIHA is associated with IgM antibodies, which are capable of fixing complement, resulting in intravascular hemolysis. AIHA is often abrupt in onset and may result in severe anemia, jaundice, and splenomegaly. Spherocytes, which are also seen in hereditary spherocytosis, are the characteristic sign on peripheral blood smear (Figure 26-5).

FIGURE 26-5 Peripheral blood smear from a patient with autoimmune hemolytic anemia demonstrating

What is the most devastating type of alloimmune hemolytic anemia?

numerous spherocytes (arrows), small round RBCs lacking central pallor, and many polychromatic RBCs. (From Weksler BB, Schechter GP, Ely SA. Wintrobe’s Atlas;of Clinical Hematology. 2nd ed. Philadelphia, PA: Wolters

Alloantibodies (antibodies directed against foreign antigens) can develop after pregnancy or exposure to blood transfusions. Most often, these alloantibodies are of the IgG class and result in delayed-onset mild extravascular hemolytic anemia. One of the most dramatic examples of alloimmunity is the acute hemolytic transfusion reaction, which occurs with transfusion of ABO-incompatible blood. Naturally

534

occurring ABO alloantibodies are typically of the IgM class, which are capable of fixing complement and causing rapid-onset massive intravascular hemolysis of thetforeign RBCs, which can lead to disseminated intravascular coagulation, renal failure, hypo ensive shock, and death. 3 What medications are most Medications can cause hemolytic anemia in a number of ways, such as triggering AIHA (eg, frequently implicated in drug- methyldopa), thrombotic microangiopathy (eg, quinine), and oxidative hemolysis, particularly in induced immune-mediated patients with G6PD deficiency (eg, dapsone). Medications can also cause an immune-mediated hemolytic anemia? hemolytic anemia by coating the RBCs, which are subsequently targeted by drug-dependent antibodies. These antibodies are usually of the IgG class and cause extravascular hemolysis. Antibiotics are the most commonly implicated agents; including cefotetan, ceftriaxone, and piperacillin. The Coombs test is positive in the majority of these patients. Hemolytic anemia resolves quickly after discontinuation of the drug. Future exposures tend to trigger more severe hemolytic anemia.9 What are the features of Hypersplenism occurs as a result of splenomegaly and describes excessive sequestration and destruction hemolytic anemia related to of RBCs. The ensuing hemolytic anemia is typically mild and associated with other cytopenias.4 hypersplenism? What is spur cell anemia? Acanthocytes are dense and contracted RBCs with irregularly spaced “thorny” projections on the surface (echinocytes are similar, but the projections are longer, more numerous, and more evenly distributed around the periphery of the cell). These cells form in patients with advanced liver disease as a result of an increase in cholesterol content and cholesterol-to-phospholipid ratio of the cell membrane. Acanthocytes evolve into spur cells that are eventually destroyed by the spleen, resulting in extravascular hemolysis of moderate to severe degree.10 What are the clinical features of PNH is a rare but life-threatening disease that affects men and women equally, and is most frequently paroxysmal nocturnal diagnosed in the fourth decade of life. A somatic mutation causes deficient production of RBC hemoglobinuria? membrane proteins, leaving cells vulnerable to complement-mediated destruction (particularly in the setting of infectious or inflammatory stimuli). Intravascular hemolysis results in hemoglobinuria that is most pronounced in the morning, a hallmark of the disease. Other common clinical manifestations include venous thrombosis and pancytopenia.3,4

535

Toxic Causes of Acquired Hemolytic Anemia

What are the toxic causes of acquired hemolytic anemia?

Hemolytic Dapsone toxicity. anemia, chocolate-colored blood, and hemoglobin oxygen saturation of 85% by pulse oximetry. Associated with Lead poisoning. microcytic anemia and basophilic stippling on the peripheral blood smear. Reptiles and Snake bites, spider bites, and bee and wasp stings. invertebrates. Liver disease, Wilson’s disease. Kayser-Fleischer rings (see Figure 17-2), and hemolytic anemia. The “gift of the Arsenic poisoning.11 Borgias.”

Which medications can cause oxidative hemolysis even in patients without G6PD deficiency? What is the mechanism of hemolytic anemia related to lead toxicity? Where is the brown recluse spider (Loxosceles reclusa) endemic in the United States? How common is hemolytic anemia in patients with Wilson’s disease? What are the features of acute arsenic poisoning?

Some medications such as nitrates, chlorates, methylene blue, dapsone, and cisplatin are capable of causing oxidative hemolysis even in the absence of G6PD deficiency.3

Lead poisoning is associated with an acquired deficiency of erythrocyte pyrimidine 5′-nucleotidase (P5′N-1), an enzyme involved in nucleotide metabolism, which causes hemolytic anemia by an unknown mechanism. Hereditary P5′N-1 deficiency also occurs but is rare. The hemolytic anemia associated with lead poisoning is mild to moderate, and may occur along with abdominal pain and peripheral edema. Basophilic stippling is the characteristic finding on peripheral blood smear (see Figure 41-5).12 The brown recluse spider is endemic to the mid-Southern and Southwestern United States. It is the most common cause of morbidity related to spider envenomation. Severe hemolytic anemia tends to develop within a week of envenomation, including features of both intravascular and extravascular hemolysis. Management is supportive, with resolution occurring over a period of several weeks.13

Wilson’s disease is a rare inherited disorder that results in the accumulation of copper to toxic levels, often manifesting with liver and neurologic disease. Hemolytic anemia develops in approximately 10% of cases, usually when liver disease is advanced and hepatocellular necrosis results in the release of copper into the bloodstream. Both intravascular and extravascular hemolysis may occur. The combination of elevated serum copper and low serum ceruloplasmin is consistent with hemolysis related to Wilson’s disease, whereas both are typically low in Wilson’s disease without hemolysis. Prognosis is poor when Wilson’s disease is not recognized and treated.14 Arsenic exposure usually occurs as a result of unintentional arsine gas inhalation (eg, in industrial electronics manufacturing), or arsenic ingestion from contaminated food or drinks. Intoxication can be acute or chronic. Intravascular hemolysis occurs within 2 to 24 hours of acute exposure and is often accompanied by Gastrointestinal sequelae (eg, abdominal pain, nausea, vomiting), and neurologic sequelae (eg, peripheral neuropathy, seizure, coma). Total urine arsenic level is most commonly used to make the diagnosis. Hair and nail analysis may also be useful.

536

Plasmapheresis may be beneficial in patients with hemolytic anemia.10,15

537

Traumatic Causes of Acquired Hemolytic Anemia

What peripheral smear finding is characteristic of traumatic causes of hemolytic anemia?

The presence of schistocytes on peripheral blood smear is the telltale sign of hemolytic anemia related to trauma. Schistocytes are fragmented RBCs that take the form of crescents, helmets, triangles, or microspheres. Hemolytic anemia related to intravascular RBC fragmentation is referred to as microangiopathic hemolytic anemia (MAHA).10

What are the causes of microangiopathic hemolytic anemia?

Hemolytic anemia, thrombocytopenia, Acute Kidney Injury, fever, and delirium. Iatrogenic. A previously healthy 24-year-old man who is training for an upcoming marathon complains of episodes of red- colored urine.

What is thrombotic microangiopathy (TMA)?

What are some examples of hardware that can cause mechanical hemolysis? What are the features of march hemoglobinuria?

Thrombotic thrombocytopenic purpura (TTP).

Intravascular hardware. Mechanical “march” hemoglobinuria.

TMA refers to a diverse group of conditions with characteristic endothelial and blood vessel wall abnormalities associated with arteriolar and capillary thrombosis, leading to MAHA, thrombocytopenia, and organ damage. The primary syndromes of TMA include TTP, HUS (most often caused by Shiga toxin–producing Escherichia coli O157:H7), and drug-induced TMA. Thrombotic microangiopathy can also occur as a result of a variety of systemic conditions, including disseminated intravascular coagulation (DIC), severe hypertension, HELLP syndrome (hemolysis, elevated liver enzymes, low platelets), systemic infection (eg, human immunodeficiency virus), malignancy, autoimmune disorders (eg, systemic lupus erythematosus, antiphospholipid syndrome), severe vitamin B12 deficiency, and hematopoietic stem cell or organ transplantation.16 Prosthetic heart valves, patches for septal defects, ventricular assist devices (eg, left ventricular assist device [LVAD]), and transjugular intrahepatic portosystemic shunt (TIPS) may cause mechanical hemolysis. The presence of turbulent blood flow is generally necessary to cause RBC fragmentation (eg, mitral regurgitation in a patient with a prosthetic mitral valve).10 March hemoglobinuria describes mechanical trauma to RBCs caused by repetitive forceful contact of the body with hard surfaces, resulting in mild self-limited episodes of hemolytic anemia. It most often occurs in young men who run competitively but may also be precipitated by activities such as marching, drumming, and karate. Intravascular hemolysis results in red- or cola-colored urine immediately after exercise, which resolves within a few hours. Chronic recurrent hemoglobinuria can lead to iron-deficiency anemia in these patients.10

538

Infectious Causes of Acquired Hemolytic Anemia

What are the general mechanisms of hemolysis caused by infection?

Infectious agents can cause hemolysis via direct invasion of RBCs or through a variety of indirect mechanisms, including immune-mediated destruction, toxin-mediated destruction, hypersplenism, and trauma-mediated destruction (ie, MAHA).17

What are the infectious causes of acquired hemolytic anemia?

A 25-year-old woman presents with fever, malaise, and hemolytic anemia after a recent trip to the Ivory Coast. Bloody diarrhea, hemolytic anemia, and Acute Kidney Injury. An anaerobic gram-positive rod. Atypical pneumonia. This organism is associated with community-acquired pneumonia and meningitis. A tick-borne disease. In 1865, a medical student named Carrión developed fatal hemolytic anemia after inoculating himself with this organism.

What are the features of the hemolytic anemia caused by malaria?

What is the mechanism of hemolysis caused by infection with enterohemorrhagic Escherichia coli? What is the mechanism of hemolysis caused by infection with Mycoplasma pneumoniae? What is the prognosis of patients with hemolytic anemia

Malaria.

Enterohemorrhagic Escherichia coli (EHEC).

Clostridium perfringens. Mycoplasma pneumoniae. Haemophilus influenzae type B.

Babesiosis. Bartonella bacilliformis (Carrión’s disease or Oroya fever).17

Malaria is a mosquito-borne disease and the most common cause of hemolytic anemia in endemic countries. Four different species are capable of causing disease in humans. Plasmodia falciparum is responsible for most cases in Africa and Southeast Asia, while Plasmodia vivax is the most frequent culprit in Central America and India. Hemolytic anemia is most often a result of splenic destruction of infected RBCs. However, acute severe intravascular hemolysis (known as blackwater fever) can occur and may be precipitated by certain medications (eg, quinine). Diagnosis of malaria can be made by direct visualization of the parasites using thin and thick blood smears.10 HUS is the most serious complication of EHEC infection. It is most often caused by serotype O157:H7 and is characterized by the triad of hemolytic anemia, thrombocytopenia, and Acute Kidney Injury. Use of antibiotics does not shorten the duration of EHEC infection, and evidence suggests antibiotics may precipitate HUS in these patients.18

In more than one-half of cases, Mycoplasma pneumoniae infection is associated with cold agglutinins (IgM immunoglobulins) that attack RBCs, fix complement, and cause intravascular hemolysis. These antibodies develop during the first 7 to 10 days of infection, peak after a few weeks, and persist for a few months. Most cases of hemolysis are not severe.17

Clostridium perfringens bacteremia is complicated by toxin-mediated severe intravascular hemolysis in up to 15% of cases. Mortality rate is approximately 75% with a median time to death of 10 hours. Survival is improved in cases where both penicillin and clindamycin are used instead of monotherapy and in patients who undergo surgical debridement of an infected focus. Hyperbaric oxygen therapy may also be effective, as Clostridium species lack

539

related to superoxide dismutase and cannot survive in oxygen-rich environments.19 Clostridium perfringens? What is the Severe hemolysis is a rare complication of invasive Haemophilus influenzae B infection. It occurs when the capsular mechanism of polysaccharide is released from the organism and binds to the surface of RBCs, which later becomes the target of hemolysis caused immune-mediated destruction, with ensuing extravascular hemolysis and complement-mediated intravascular by infection with hemolysis.17 Haemophilus influenzae type B? What finding on Like malaria, Babesia parasites directly invade RBCs and alter structure and function, leading to splenic sequestration peripheral blood and destruction. Peripheral blood smear examination may reveal intracellular organisms. Although uncommon, the smear is pathognomonic for presence of tetrads of merozoites that resemble a “Maltese cross” is pathognomonic for babesiosis 
(Figure 26-6).3,17 babesiosis?

FIGURE 26-6 Although the Babesia species can demonstrate ring forms and be confused with Plasmodia falciparum, the

What are the features of Bartonella bacilliformis infection?

presence of the “Maltese cross,” comprised of tetrads of Babesia merozoites, can be a distinguishing feature. (From Pereira I, George TI, Arber DA. Atlas of Peripheral Blood: The Primary Diagnostic Tool. Philadelphia, PA: Wolters Kluwer Health; 2012.)

Bartonella bacilliformis is endemic to Peru, Ecuador, and Colombia and is transmitted via sand fly bite. The organisms directly invade RBCs, usually resulting in acute-onset intravascular and extravascular hemolysis with severe and life- threatening anemia. The peripheral blood smear may reveal intraerythrocytic bacilli.3,17

540 Case Summary dyspnea, andldark tea-colored urine and is found to have hemolytic A previous y healthy 31-year-old woman presents with fatigue, anemia in the setting of a spider bite and ingestion of fava beans.

What is the most likely cause of hemolytic anemia in this patient? Glucose-6-phosphate dehydrogenase deficiency.

541

Bonus Questions

G6PD deficiency? from Africa, the Mediterranean region, Southeast Asia, and Oceania, where it is relatively protective against malaria.iInheritance is X-linked, but heterozygous deficiency causeia? antioxidant compounds within the RBC. When G6PD is deficient,tRBCs are susceptible to oxidativeistress, which results in premature lysis. The vast majority ofral How common is G6PD deficiency is the most frequent RBC enzyme defect, occurring in up to one-fifth of the general population in some regions. It is most common in patients females can be affected to the same degree as hemizygous males because of the phenomenon of X-chromosome inact vation. 3 Why does G6PD G6PD is an enzyme found in RBCs that is important in producing reduced nicotinamide adenine dinucleotide phosphate (NADPH), which acts to stabilize seve hemolytic anem patients are asymptomatic. However, patients can experience acu e episodes of moderate to severe ntravascular and extravascular hemolytic anemia when exposed to oxidative agents (such as fava beans via direct ingestion of beans or exposure to pollen from the fava plant), infections, or medications (eg, sulfamethoxazole).3 Why does ingestion Divicine and isouramil are thought to be the components of fava beans responsible for the increase in RBC oxidant stress. Not all G6PD-deficient individuals of fava beans cause experience favism. hemolytic anemia in It is said that Greek mathematician and philosopher Pythagoras, when chased by his enemies, chose certain death over fleeing through a field of fava beans.20 patients with G6PD deficiency? What features of this Features of this case consistenttwith G6PD deficiency include the dark tea-colored urine, whichtsuggests intravascular hemolysis, the recent exposure to fava Why is the peripheral For a number of reasons, the peripheral blood smear is important in the diagnosis of G6PD deficiency, particularly during an acute attack of hemolytic anemia. G6PD deficiency? case are suggestive of beans, and the presence of a bi e cell on the peripheral blood smear (see Figure 26-1, arrow). Re iculocytes are also present (see Figure 26-1, arrowheads). blood smear First, the results of a peripheral blood smear evaluation return sooner than the G6PD activity assay. Second, G6PD activity may be normal during acute episodes. importantiin the? For example, in heterozygous women, acute hemolysis preferentially involves abnormal RBCs, leaving a higher proportion of normal cells in circulation, leading G6PD deficiency investigat on of to a falsely normal assay. 21 What is the relevance Envenomation by the brown recluse spider can cause hemolytic anemia. However, in this case, that possibility is unlikely because the brown recluse is not of the spider bite in endemic to Oregon. this case? deficiency managed? Diagnosis and prevention of future episodes are important for long-term management. Patients should be given a list of typical triggerslwith advice totavoid How is G6PD The treatment for acute hemolytic anemia associated with G6PD deficiency is mostly supportive, but can include blood transfusions if c inically indica ed. exposure when possible. 3

542 Key Points

anemia occurs when hemolysis outpaces bone marrow production. Hemolysis is the premature destruction of RBCs; hemolytic The symptoms of hemolytic anemia include fatigue, loss of stamina, and dyspnea. Dark cola-colored urine can indicate

intravascular or severe extravascular hemolysis, red-colored urine can indicate rapid intravascular hemolysis, and flank pain can be present in abrupt-onset hemolysis. Phys cal findings of hemolytic anemia include fever, tachycardia,

wideipulse pressure, forceful heartbeat, strong peripheral pulses, and jaundice and splenomegaly (in extravascular hemolysis). hyperproliferative anemia associated with evidence of RBC Hemoglobinemia and hemoglobinuria are the biochemical patients with hemolytic anemia and can betsuggestive of specific systolic flow murmur, pallor of the mucous membranes and skin, Hemolytic anemia is typically a normocytic or macrocytic destruction (eg, elevated serum LDH). hallmarks of intravascular hemolysis. The peripheral blood smear is an importan diagnostic study in etiologies. Hemolytic anemia can be inherited or acquired.

Inherited hemolytic anemia can be caused by defects that involve Acquiredthemolytic anemia can be immunologic, toxic, traumatic, hemoglobin, intracellular enzymes, or the membrane-
 cytoskele on complex. or infectious.

543

References 1. Eadie GS, Brown IW Jr. The potential lifeispan and ultimate survival of fresh red blood cells in normal healthy recipients as stud ed by simultaneous Cr51 tagging and differential hemolysis. J Clin Invest. 1955;34(4):629-636.

  1. Shemin D, Rittenberg D. The life span of the human red blood cell. J Biol Chem. 1946;166(2):627-636.

  2. Longo DL, Fauci AS, Kasper DL, Hauser SL, Jameson JL, Loscalzo J, eds. Harrison’s Principles of Internal Medicine. 18th ed. New York, NY: McGraw-Hill; 2012.

  3. Guillaud C, Loustau V, Michel M. Hemolytic anemia in adults: main causes and diagnostic procedures. Expert Rev Hematol. 2012;5(2):229-241.

  4. Aslinia F, Mazza JJ,;Yale SH. Megaloblastic anemia and other causes of macrocytosis. 6. Peters M, Heijboer H, Smiers F, Giordano PC. Diagnosis and management of Clin Med Res. 2006 4(3):236-241. thalassaemia. BMJ. 2012;344:e228.

  5. Fairhurst RM, Casella JF. Images in clinical medicine. Homozygous hemoglobin C disease. N Engl J Med. 2004;350(26):e24.

  6. Nagel RL, Fabry ME, Steinberg MH. The paradox of hemoglobin SC disease. Blood Rev. 2003;17(3):167-178.

  7. Garratty G. Drug-induced.immune hemolytic anemia. Hematology Am Soc Hematol 10. Greer JP, Foerster J, Rodgers GM, et al, eds. Wintrobe’s Clinical Hematology. 12th ed. Educ Program. 2009:73-79 Philadelphia, PA: Lippincott Williams & Wilkins, A Wolters Kluwer Business; 2009.

  8. Meek WJ. The gentle art of poisoning. J Am Med Assoc. 1955;158(4):335-339. 12hemolytic anemia, basophilic stippling, erythrocyte pyrimidine 5’-nucleotidase ith . Valentine WN, Paglia DE, Fink K, Madokoro G. Lead poisoning: association w deficiency, and intraerythrocytic accumulation of pyrimidines. J Clin Invest.

1976;58(4):926-932. 13envenomation leading to acute.hemolytic anemiaiin six adolescents. J Pediatr. . McDade J, Aygun B, Ware RE Brown recluse sp der (Loxosceles reclusa) 2010;156(1):155-157.

  1. Walshe JM. The acute haemolytic syndrome in Wilson’s disease–a review of 22 patients. QJM. 2013;106(11):1003-1008.

  2. Lee;JJ, Kim YK, Cho SH, et al. Hemolytic anemia as a sequela.of arsenic intoxication following long-term ingestion of traditional Chinese medicine J Korean Med Sci. 2004 19(1):127-129.

  3. George JN, Nester.CM. Syndromes of thrombotic microangiopathy. N Engl J Med. 2014;371(7):654-666
  4. McCullough J. RBCs)as targets of infection. Hematology Am Soc Hematol Educ Program. 2014;2014(1 :404-409.
  5. Page AV, Liles WC. Enterohemorrhagic Escherichia coli infections and the hemolytic- uremic syndrome. Med Clin North Am. 2013;97(4):681-695, xi. . Simon TG, Bradley J, Jones A, Carino G. Massive intravascular hemolysis from

19Clostridium perfringens septicemia: a review. J Intensive Care Med. 2014;29(6):327-333. 20. Beutler E. Glucose-6-phosphate dehydrogenase deficiency: a historical perspective. Blood. 2008;111(1):16-24.

  1. Bain BJ. Diagnosis from the blood smear. N Engl J Med. 2005;353(5):498-507.

544

545

CHAPTER 27

546

Pancytopenia

547 Case: A 34-year-old man with easy bruising anA 34-year-old man presents with several weeks of fatigue, dyspnea, Northwest. He.does not take any prescription or over-the-counter d easy bruising. The patient grew up in Belarus, near its southeastern border with Ukraine, before moving to Oregon at 14 years of age He has not recently traveled outside of the Pacific medications or supplements. He does not use illicit drugs. is The patient is afebrile with a heart rate of 96 beats per minute. There conjunctival pallor. No lymphadenopathy or organomegaly is appreciated. Peripheral white blood cell (WBC) count is 2.1 K/µL, hemoglobin is

7.6 g/dL, mean corpuscular volume (MCV) is 93 fL, and platelet count is 28 K/µL. Plasma methylmalonic acid and homocysteine levels are within normal limits. Human immunodeficiency virus (HIV)

antibodies are not detected in the serum. Peripheral blood smear reticulocytopenia. Bone marrow aspirate shows hypocellularity with hematoxylin and eosin staining is shown in Figure 27-1. demonstrates leukopenia, thrombocytopenia, and anemia with normal cellular morphology. A core biopsy of the bone marrow with

FIGURE 27-1

What is the most likely cause of pancytopenia in this patient?

What is pancytopenia? Where are blood cells produced?

Pancytopenia describes the simultaneous presence of leukopenia, anemia, and thrombocytopenia. Hematopoietic stem cells found within bone marrow give rise to myeloid and lymphoid progenitor cells. These cells further differentiate into WBCs, red blood cells (RBCs), and platelets (Figure 27-2). These cells are eventually cleared by the reticuloendothelial system. WBCs survive for hours to years, depending on type; RBCs survive for 120 days; and platelets survive for 9 days.1-3

548

What are the symptoms of pancytopenia? What are the physical findings of pancytopenia? What is the role of the peripheral blood smear in the investigation of pancytopenia? What is the role of bone marrow evaluation in the investigation of pancytopenia?

What are the general mechanisms of pancytopenia?

FIGURE 27-2 Hematopoiesis. (From Mc Connell TH. The Nature of Disease Pathology for the Health Professions. 2nd ed. Philadelphia: Lippincott Williams & Wilkins; 2007.)

Symptoms of pancytopenia occur as a result of the individual cytopenias (usually anemia and thrombocytopenia) and may include generalized weakness (most common), dyspnea, chills, weight loss, easy bruising, and easy or prolonged bleeding. Patients may also report frequent infections (eg, upper respiratory tract infection).4 Physical findings of pancytopenia occur as a result of the individual cytopenias (usually anemia and thrombocytopenia) and may include fever, pallor (most common), splenomegaly, hepatomegaly, jaundice, petechial rash, and lymphadenopathy.4 The peripheral blood smear can confirm the presence of pancytopenia and may identify a specific underlying etiology based on cell morphology (eg, megaloblastosis in patients with vitamin B12 deficiency, blast cells in patients with acute leukemia).

Bone marrow aspiration and biopsy are not always necessary in patients with pancytopenia, such as when the underlying etiology is strongly suggested by the history, physical examination, and other laboratory studies, and a primary bone marrow disorder is not suspected (eg, pancytopenia in a patient who recently received cytotoxic chemotherapy). However, when the underlying etiology remains elusive or the pancytopenia is persistent, a bone marrow examination is often helpful. The aspirate may demonstrate abnormal cellular morphology not evident on peripheral blood smear, and the biopsy can determine bone marrow cellularity (hypocellular, normocellular, hypercellular) (Figure 27-3) and identify infiltrative processes (eg, fibrosis).5,6

FIGURE 27-3 Bone marrow biopsy from an otherwise healthy 60-year-old man demonstrates normal cellularity (40%-50%). (From McClatchey KD. Clinical Laboratory Medicine. 2nd ed. Philadelphia: Lippincott Williams & Wilkins; 2002.)

Pancytopenia can develop as a result of bone marrow hypoplasia (ie, hypocellularity), ineffective hematopoiesis, bone marrow infiltration, or hypersplenism.

549

550

Pancytopenia Related to Bone Marrow Hypoplasia

What is Hypoplastic bone marrow describes an abnormally low proportion of hematopoietic stem cells in the absence of an bone infiltrative process such as myelofibrosis. The cells are replaced by adipose tissue. Normal bone marrow cellularity in adults marrow ranges from 40% to 60%, with a slight decrease in the elderly. Bone marrow cellularity can be established with biopsy (see hypoplasia? ).7 Figure 27-3 Aplastic anemia is defined as the combination of pancytopenia and bone marrow hypoplasia. However, acute and transient causes of pancytopenia and bone marrow hypoplasia (eg, cytotoxic chemotherapy) are generally not referred to as aplastic anemia. If the cytopenias and hypoplasia are not reversible, then acquired aplastic anemia is an appropriate description.

What are the causes of bone marrow hypoplasia?

A 41-year-old woman develops Antithyroid medication (eg, methimazole, propylthiouracil) for hyperthyroidism. pancytopenia after starting therapy for a condition characterized by weight loss, heat intolerance, and tremor. A 56-year-old man with recent-onset hair Arsenic poisoning. loss, peripheral neuropathy, and pancytopenia has had recurrent hospital admissions for abdominal pain, vomiting, diarrhea, and delirium, which seem to occur only after consuming meals prepared by his wife. A 46-year-old man presents with fever, Parvovirus B19 infection. symmetric polyarticular inflammatory arthritis, pancytopenia, and bone marrow biopsy showing giant pronormoblasts with inclusions. Women of reproductive age. Pregnancy. Hemolysis, pancytopenia, and thrombosis. Paroxysmal nocturnal hemoglobinuria (PNH). Congenital. Inherited aplastic anemia. Failure to identify a cause of acquired bone Idiopathic aplastic anemia. marrow hypoplasia and pancytopenia despite a complete workup.

Which medications are associated with bone marrow hypoplasia and pancytopenia?

Numerous medications are associated with bone marrow hypoplasia and pancytopenia, including antibiotics (eg, chloramphenicol), chemotherapeutic agents (eg, doxorubicin), antithyroid agents (eg, methimazole), nonsteroidal anti-inflammatory drugs (eg, indomethacin), anticonvulsants (eg, carbamazepine), and lithium. In some cases the nature of the hypoplasia is transient, resolving within days to weeks of withdrawing the offending agent. However, in other cases, the hypoplasia may persist, resulting in acquired aplastic anemia.8,9

Which toxins are associated with bone marrow hypoplasia and pancytopenia? Which infections are associated with bone marrow hypoplasia and pancytopenia?

Toxic causes of bone marrow hypoplasia and pancytopenia include alcohol, radiation exposure (both iatrogenic and environmental), benzene, arsenic, and insecticides. Alcohol-related pancytopenia is typically reversed by reducing or abstaining from alcohol consumption. Heavy alcohol consumption is also associated with folate deficiency (ie, megaloblastic anemia) and liver disease with splenomegaly, which may contribute to pancytopenia. Other toxic causes of marrow hypoplasia and pancytopenia may persist despite removal of the toxin, resulting in acquired aplastic anemia.9,10 Overwhelming bacterial sepsis is associated with pancytopenia. The mechanisms are multifactorial, including bone marrow hypoplasia from the infection itself, medications used to treat the infection (eg, antimicrobials), and disseminated intravascular coagulation. Other infectious agents associated with bone marrow hypoplasia include seronegative hepatitis (possibly an infectious agent that has yet to be identified), Epstein-Barr virus, cytomegalovirus, HIV, parvovirus B19, miliary tuberculosis (TB), dengue virus, and leptospirosis. Infection can also cause pancytopenia by triggering the life-threatening hemophagocytic lymphohistiocytosis (HLH) syndrome. In these patients, bone marrow examination may demonstrate hemophagocytosis.9,11

551

What are the characteristics of bone marrow hypoplasia related to pregnancy? What is the relationship between paroxysmal nocturnal hemoglobinuria and aplastic anemia? What are the inherited causes of aplastic anemia?

Bone marrow hypoplasia and pancytopenia can develop at any point during pregnancy, and are typically progressive before resolving after abortion or delivery. Patients with true aplastic anemia are more likely to suffer relapse during pregnancy. Supportive care is the mainstay of treatment for these patients; transfusion with blood products (eg, platelets) may be necessary. Cyclosporine is safe during pregnancy and should be considered for patients who require frequent transfusions.12 A significant portion of patients with PNH will develop bone marrow hypoplasia and pancytopenia. Conversely, up to one-half of patients with acquired aplastic anemia have small PNH clones that can be detected by flow cytometry of peripheral blood. Over time, these clones can remain stable, progress, or regress in size. Clinically significant clones may lead to the manifestations of “classic” PNH, which consists of clinical or laboratory evidence of intravascular hemolysis. The treatment for aplastic anemia is not affected by the presence of PNH.9,12

Aplastic anemia is associated with several rare genetic disorders, including Fanconi anemia, dyskeratosis congenita, Shwachman-Diamond syndrome, and congenital amegakaryocytic thrombocytopenia. Inherited aplastic anemia is usually diagnosed in childhood but sometimes presents in adulthood. It is important to consider these conditions in any adult newly diagnosed with aplastic anemia. Family history and observation of extrahematopoietic abnormalities can provide clues to an underlying genetic condition (eg, short stature, skin hyper/hypopigmentation, and skeletal abnormalities are suggestive of Fanconi anemia) (Figure 27-4).9,12

FIGURE 27-4 Hands of a patient with Fanconi anemia. Bilateral thumb hypoplasia is present. Other features include skin

What proportion of aplastic anemia cases are idiopathic?

pigmentation changes, short stature, upper limb abnormalities, renal malformations, ophthalmologic problems, hypogonadism, and cardiac malformations. (Courtesy of Dr. I. Quirt.)

Most cases of aplastic anemia are idiopathic in nature. Patients with bone marrow hypoplasia and pancytopenia must undergo a thorough workup to rule out alternative causes before a diagnosis of idiopathic aplastic anemia is given.12

552

Pancytopenia Related to Ineffective Hematopoiesis

What is ineffective hematopoiesis?

Ineffective hematopoiesis describes the failure to produce mature blood cells as a result of dysfunctional progenitor cells, despite the presence of normo- or hypercellular bone marrow.

What are the causes of ineffective hematopoiesis?

A 43-year-old woman who follows a strict vegan diet presents with ataxia, peripheral neuropathy, and pancytopenia with oval macrocytosis and hypersegmented neutrophils on the peripheral blood smear. Usually a disease of the elderly, associated with macrocytosis.

What are the causes of megaloblastic anemia?

What are the features of pancytopenia caused by myelodysplasia?

Vitamin B12 deficiency.

Myelodysplasia.

Megaloblastic anemia may be caused by vitamin B12 deficiency, folate deficiency, copper deficiency, and drugs or toxins. It is the most common cause of pancytopenia in the developing world. Peripheral blood smear usually has characteristic features (see Figure 25-5). Bone marrow aspiration and biopsy demonstrate hypercellularity with evidence of abnormal proliferation and maturation of multiple myeloid cell lines, including large, oval-shaped erythroblasts that contain a characteristic finely stippled, lacy nuclear chromatin pattern surrounded by normal-appearing cytoplasm (known as nuclear- cytoplasmic dissociation).9,13 Myelodysplastic syndrome refers to a group of conditions characterized by bone marrow failure as a result of abnormal cellular maturation. Cytopenias may occur individually or in combination, usually starting with anemia. Pancytopenia is more likely to be a feature of high-grade myelodysplastic syndromes. Bone marrow failure is typically progressive, and may evolve to acute myeloid leukemia. The bone marrow is normo- or hypercellular in most patients with myelodysplasia, however one-fifth of cases present with bone marrow hypoplasia that may be difficult to distinguish from aplastic anemia.9,14

553

Pancytopenia Related to Bone Marrow Infiltration

What is bone marrow infiltration?

Bone marrow infiltration describes the replacement of hematopoietic stem cells with nonadipose tissue. This leads to hematopoiesis that occurs outside of the bone marrow (ie, extramedullary hematopoiesis) in locations such as the liver and spleen, which causes hepatosplenomegaly. Extramedullary hematopoiesis is associated with characteristic peripheral blood smear findings, including teardrop-shaped RBCs and leukoerythroblastosis (ie, nucleated RBCs and immature WBCs) (Figure 27-5).

FIGURE 27-5 Peripheral blood smear from a patient with myelofibrosis demonstrating characteristic findings, including teardrop- shaped RBCs (arrow), nucleated RBCs (arrowhead), large platelets (small arrows), and immature WBCs (asterisk). (From Greer JP, Arber DA. Wintrobe’s Clinical Hematology. 13th ed. Philadelphia, PA: Lippincott Williams and Wilkins; 2014.)

What are the causes of bone marrow infiltration?

Blasts on peripheral blood smear. A 39-year-old man from India with a history of HIV presents with fever, dyspnea, and weight loss, and is found to have pancytopenia and innumerable tiny densities throughout the lungs on chest imaging. Attempted aspiration of the bone marrow results in a “dry” tap.

Which malignancies can cause pancytopenia?

What are the features of pancytopenia caused by miliary tuberculosis?

What is myelofibrosis?

Acute leukemia. Miliary TB.

Myelofibrosis.

Acute myeloid leukemia is among the most common causes of pancytopenia in adults. The bone marrow is hypercellular and replaced with blasts, and the peripheral blood smear may also show blasts. Other malignancies associated with pancytopenia include non-Hodgkin’s lymphoma, chronic leukemia (most often hairy cell leukemia), multiple myeloma, and nonhematologic malignancies that metastasize to the bone marrow.9 Pancytopenia develops in a minority of patients with miliary TB, usually occurring in those with HIV. Common clinical manifestations include fever, respiratory distress, splenomegaly, and lymphadenopathy. Classic miliary nodules are frequently present on chest imaging. Bone marrow biopsy is associated with a high diagnostic yield and typically demonstrates caseating granulomas. Pancytopenia in patients with miliary TB is associated with a poor prognosis even with treatment. Other infections that can invade the bone marrow and cause pancytopenia include fungi and brucellosis.15,16 Fibrosis of the bone marrow can develop as a primary process (primary myelofibrosis) or secondary process (myelophthisis). Conditions that are most often associated with myelophthisis include mycobacterial infection, fungal infection, HIV, sarcoidosis, invading malignancy, lysosomal storage disorders, and external beam radiation therapy. Bone marrow aspiration is often unsuccessful (producing a “dry” tap). On biopsy, the bone marrow is initially hypercellular before becoming hypocellular with Increased reticulin or collagen fibrosis.14

554

555

Pancytopenia Related to Hypersplenism

What is hypersplenism?

What are the mechanisms of pancytopenia related to splenomegaly?

Hypersplenism is characterized by splenomegaly and one or more peripheral cytopenias in the setting of normo- or hypercellular bone marrow. When hypersplenism is the sole driver of pancytopenia, splenectomy is often curative. Splenomegaly is frequently associated with conditions that cause pancytopenia independently (eg, myelofibrosis) and may contribute to the severity of the cytopenias.14 Pancytopenia in patients with splenomegaly occurs via a variety of mechanisms, including sequestration of blood cells within the spleen, premature destruction of blood cells, and an increase in plasma volume (which can cause pancytopenia via hemodilution). Massive spleens are capable of sequestering up to 90% of peripheral platelets, 65% of granulocytes, and 30% of RBCs.9,17

What are the causes of hypersplenism?

A 54-year-old man with a history of chronic hepatitis C infection develops jaundice, spider angiomas, ascites, and pancytopenia. Splenomegaly, diffuse lymphadenopathy, and elevated serum lactate dehydrogenase. A parasitic infection endemic to some parts of the world that usually presents with months of fatigue, fever, weight loss, and splenomegaly. A 46-year-old woman with polyarticular inflammatory arthritis, neutropenia, and splenomegaly. An inborn error of metabolism characterized by the accumulation of glucocerebroside within macrophage lysosomes.

What are the causes of congestive splenomegaly?

Which hematologic disorders are associated with splenomegaly? Which infections are associated with splenomegaly?

Which autoimmune diseases are associated with splenomegaly? Which infiltrative diseases are associated with splenomegaly?

Cirrhosis.

Lymphoma. Visceral leishmaniasis (kala-azar).

Felty syndrome (ie, neutropenia and splenomegaly in patients with rheumatoid arthritis).

Gaucher disease.

Congestive splenomegaly can develop with any cause of portal hypertension, including constrictive pericarditis, right-sided heart failure, hepatic vein obstruction, liver disease, portal vein obstruction, and splenic vein obstruction.18 Hematologic disorders associated with splenomegaly include primary myelofibrosis, chronic leukemia, indolent lymphoma, polycythemia vera, hairy cell leukemia, and thalassemia.18 Splenomegaly can be associated with viral infection (eg, Epstein-Barr virus, cytomegalovirus, viral hepatitis, HIV), bacterial infection (eg, endocarditis, enteric fever, TB, brucellosis), parasitic infection (eg, malaria, visceral leishmaniasis, schistosomiasis), and fungal infection (eg, histoplasmosis).18 Autoimmune diseases associated with splenomegaly include systemic lupus erythematosus, rheumatoid arthritis (part of the triad of Felty 
syndrome), and sarcoidosis.18 Infiltrative diseases associated with splenomegaly include Gaucher disease, Niemann-Pick disease, amyloidosis, and hemophagocytic lymphohistiocytosis.18

556 Case Summary A 34-year-old man presents with fatigue, dyspnea, and easy

bruising and is found to have pancytopenia with an abnormal bone marrow examination.

What is the most likely cause of pancytopenia in this patient? Aplastic anemia.

557

Bonus Questions

What significant finding The bone marrow biopsy in thisicase (see Figure 27-1) demonstrates hypocellularity with only small areas containing maturing cells; the majority of the bone marrow biopsy in this is present on the bone marrow is made up of adipose t ssue. Normal bone marrow cellularity depends on age; in adults it is 40% to 60% with a slight decrease in the elderly. 7 case? What is aplastic anemia? Aplastic anemia is a type of bone marrow failure characterized by the combination of bone marrow hypoplasia and pancytopenia in the absence of an years infiltrative process. It is a rare disease with an incidence of 2 to 3 per million in the industrialized world. Distribution is biphasic, with peaks at 10 to 25 and >60 years of age. Disease severity is based on the degree of bone marrow hypocellularity and peripheral cytopenias. What are the causes of Acquired aplastic anemia describes any form of secondary bone marrow hypoplasia and pancytopenia that does not resolve after removal of the nciting 12 acquired aplastic anemia? agent. In most cases, aplastic anemia develops in the absence of an identifiable inciting agent (ie, idiopathic aplastic anemia). The pathogenesis ofiidiopathic aplastic anemia is not known, but there is evidence suggesting it is immune-mediated. What defin tive treatmen Untreated severe aplastic anemia is associated with a 1-year mortality rate >80%. First-line definitive treatment options include immunosuppressive therap 12 options areiavailable for t (eg, antithymocyte globulin, cyclosporine, glucocorticoids) and allogeneic hematopoietic stem cell transplantation. The initial therapy of choice depends ony aplastic anemia? patient-specific factors (eg, age, comorbidities, donor availability, preference), and disease-specific factors (eg, severity). Other treatment options such as blood transfusions are supportive and aim to improve quality of life.12,19 near the Belarusian border there is no data to suggest that the Chernobyl disaster has impacted the incidence of aplastictanemia. However, it is possible that a relationshiptbetween the What important historical The Chernobyl disaster of 1986 resulted in the release of massive amounts of radioactive par icles into the atmosphere over a large geographical area. event occurred in the 1980s Beginning in the 1990s the incidence of thyroid cancer dramatically Increased in children exposed to radiation from the Chernobyl disaster. At his time, with Ukraine? two is established in the future. 20,21

558 Key Points

Pancytopenia describes the simultaneous presence of leukopenia, eventually cleared by the reticuloendothelial system. and dyspnea, chills, weight loss, easy bruising, and easy or prolonged splenomegaly, hepatomegaly, jaundice, petechial rash,,and Pancytopenia can develop as a result of bone marrow hypoplasia, hypersplenism. anemia, and thrombocytopenia. Mature blood cells are developed in the bone marrow Symptoms of pancytopenia include generalized weakness, bleeding. Physical findings of pancytopenia include fever, pallor lymphadenopathy. ineffective hematopoiesis, bone marrow infiltration, 
or Bone marrow hypoplasia describes an abnormally low proportion of hematopoietic stem cells in the absence of an infiltrative process. Some causes of hypoplasia can be anticipated and are reversible (eg, cytotoxic chemotherapy).

Irreversible hypoplasia is referred to as aplastic anemia. It can be Ineffective hematopoiesis is the failure to produce mature blood inherited or acquired. cells as a result of dysfunctional progenitor cells, despite the presence of normo- or hypercellular bone marrow. Bone marrow nfiltration describes the replacement of

hematopoieticistem cells with nonadipose tissue, leading to Hypersplenismtis characterized by splenomegaly and peripheral extramedullary hematopoiesis. cytopenia(s) in he setting of normo- or hypercellular bone marrow.

559

References 1. Eadie GS, Brown IW Jr. The potential lifeispan and ultimate survival of fresh red blood cells in normal healthy recipients as stud ed by simultaneous Cr51 tagging and differential hemolysis. J Clin Invest. 1955;34(4):629-636.

  1. Leeksma CH, Cohen JA. Determination of the life of human blood platelets using labelled diisopropylfluorophosphanate. Nature. 1955;175(4456):552-553.

  2. Shemin D, Rittenberg D. The life span of the human red blood cell. J Biol Chem. 1946;166(2):627-636.

  3. Gayathri BN, Rao KS. Pancytopenia: a clinico hematological study. J Lab Physicians. 2011;3(1):15-20. Devitt KA, Lunde JH, Lewis MR. New onset pancytopenia in adults: a review of

  4. underlying pathologies and their associated clinical and laboratory findings. Leuk Lymphoma. 2014;55(5):1099-1105.

  5. Weinzierl EP, Arber DA. Bone marrow evaluation in new-onset pancytopenia. Hum Pathol. 2013;44(6):1154-1164.

  6. Al-Adhadh AN, Cavill I. Assessment of cellularity in bone marrow fragments. J Clin Pathol. 1983;36(2):176-179.

  7. Laboratory studies in drug-induced pancytopenia. Br Med J. 1980;280(6212):429-430. 9. Weinzierl EP, Arber DA..The differential diagnosis and bone marrow evaluation of 10. Nakao S, Harada M, Kondo K, Mizushima N, Matsuda T. Reversible bone marrow new-onset pancytopenia Am J Clin Pathol. 2013;139(1):9-29. hypoplasia induced by alcohol. Am J Hematol. 1991;37(2):120-123.

  8. Jain A, Naniwadekar M. An etiological reappraisal of pancytopenia – largest series reported to date from a single tertiary care teaching hospital. BMC Hematol.

2013;13(1):10. 12. Killick SB, Bown N, Cavenagh J, et al. Guidelines for the diagnosis and management of adult aplastic anaemia. Br J Haematol. 2016;172(2):187-207.

  1. Aslinia F, Mazza JJ, Yale SH. Megaloblastic anemia and other causes of macrocytosis. Clin Med Res. 2006;4(3):236-241.

  2. Longo DL, Fauci AS, Kasper DL, Hauser SL, Jameson JL, LoscalzolJ, eds. Harrison’s 15severe complication of miliary tuberculosis. Rev Mal Respir. 2013;30(1):33-37. ia: a 16. Maartens G, Willcox PA, Benatar SR. Miliary tuberculosis: rapid diagnosis, Principles of Internal Medicine. 18th ed. New York, NY: McGraw-Hil ; 2012. . Achi HV, Ahui BJ, Anon JC, Kouassi BA, Dje-Bi H, Kininlman H. Pancytopen hematologic abnormalities, and outcome in 109 treated adults. Am J Med.

1990;89(3):291-296. 17Mechanism of dilutional anemia in massive splenomegaly.lBlood. 1976;47(4):629-644. 18. Elmakki. Hypersplenism: review article. J Biol Agric Healthc. 2012;2(10). . Hess CE, Ayers CR, Sandusky WR, Carpenter MA, Wetze RA, Mohler DN. 19. Young NS. Aplastic anaemia. Lancet. 1995;346(8969):228-232. 20. Hatch M, Ron E, Bouville A, Zablotska L, Howe G. The Chernobyl disaster: cancer following the accident at the Chernobyl nuclear power plant. Epidemiol Rev.

2005;27:56-66. 21. Reiners C. Radioactivity and thyroid cancer. Hormones (Athens). 2009;8(3):185-191.

560

561

CHAPTER 28

562

Platelet Disorders

563 Case: A 39-year-old man with fever and skin rash A previously healthy 39-year-old man presents to the emergency complains of a skin rash on his lower extremities and intermittent When his wife noticed confusion, she brought him to the emergency department with fatigue and malaise over the past 5 days. He also fever. He began bleeding from his gums on the day of presentation. department. Temperature is 37.7°C. There is scleral icterus and subconjunctival erythematous macules on the lower extremities. Splenomegaly is not pallor. There are innumerable 0.5 to 12 mm nonblanching appreciated. Peripheral white blood cell count is 8 K/µL, hemoglobin is 6.2 g/dL, and platelet count is 6 K/µL The corrected reticulocyte count is 18%

(reference range 0.5%-1.5%).and the immature platelet fraction is 22% bilirubin is 3.1 mg/dL with an indirect component of 2.8 mg/dL. (reference range 1%-7.5%). Serum creatinine is 2.6 mg/dL and total Peripheral blood smear is shown in Figure 28-1.

FIGURE 28-1 (Courtesy of Michael J. Cascio, MD.)

What is the most likely cause of thrombocytopenia in this patient?

How is platelet production normally regulated? What is the primary function of platelets?

Platelets are generated from megakaryocytes, which are produced in the bone marrow via the myeloid progenitor cells (see Figure 27-2). The principal regulator of platelet production is the hormone thrombopoietin, which is synthesized in the liver. Under normal circumstances, decreases in platelet and megakaryocyte mass stimulate the production of thrombopoietin. The average platelet survives for 9 days before being removed by the reticuloendothelial system.1,2 Platelets are important for maintaining the integrity of the vascular system. When there is injury to a blood vessel, endothelial cells release von Willebrand factor (VWF), which facilitates platelet adherence to the exposed collagen matrix. Bound platelets become activated and secrete adenosine diphosphate (ADP) and thromboxane A2. These substances promote further platelet aggregation and the formation of a platelet plug, achieving primary hemostasis (Figure 28-2). The coagulation cascade produces fibrin that reinforces the developing thrombus.3

564

FIGURE 28-2 Platelet adhesion, secretion, and aggregation. Von Willebrand factor mediates platelet adhesion to the

What are the clinical manifestations of platelet disorders? What are petechiae and purpura? Which laboratory tests can be used to detect platelet disorders?

What are the 2 general categories of platelet disorders?

What is the difference between qualitative and quantitative platelet disorders? Which laboratory tests are helpful in determining whether platelet dysfunction is qualitative or quantitative?

subendothelium by binding to both exposed subendothelial collagen and the platelet membrane glycoprotein Ib (GPIb). Platelet adhesion also occurs directly through platelet membrane collagen receptors, such as GPVI. Platelet adhesion leads to platelet activation and secretion of soluble platelet factors, including ADP and thromboxane A2 (TxA2), which facilitate platelet recruitment and aggregation. Platelet aggregation occurs when fibrinogen cross-links platelets by binding to GPIIb-IIIa receptors on platelet membranes.

Clinical manifestations of platelet disorders may include easy bruising, excessive bleeding, and petechial or purpuric rash. Bleeding tends to occur in the skin and mucosa (eg, gums, nasal mucosa). In contrast, coagulation disorders tend to present with ecchymoses or hemarthrosis. Some conditions may affect both platelets and the coagulation cascade (eg, von Willebrand disease), resulting in a mixed clinical picture.3 Petechiae and purpura are the result of extravasation of blood from the vasculature into the skin or mucosa, usually occurring in the dependent regions of the body. Petechiae are pinpoint hemorrhages ≤2 mm in size, purpura are 2 mm to 1 cm in size, and ecchymoses are >1 cm in size. These lesions do not blanch with pressure.3 A complete blood count (CBC), peripheral blood smear, prothrombin time/international normalized ratio (PT/INR), and activated partial thromboplastin time (aPTT) are helpful in the initial evaluation of patients with a suspected bleeding disorder. The PT/INR and aPTT are helpful in identifying coagulation defects. Bleeding time is the classic diagnostic test used to identify dysfunctional platelets but has largely been replaced by instruments that measure platelet-dependent coagulation under flow conditions, such as the Platelet Function Analyzer (PFA-100). Some conditions may affect both platelets and the coagulation cascade, resulting in a mixed laboratory picture.2,4 Platelet disorders can be qualitative or quantitative.

In qualitative platelet disorders, platelets are normal in number but abnormal in function. In quantitative platelet disorders, platelets are normal in function but abnormal in number.

The CBC and peripheral smear are useful for identifying a quantitative platelet disorder. If the platelet count is normal, then the bleeding time or PFA-100 can be used to identify a qualitative disorder.

565

Qualitative Platelet Disorders

What are the 3 mechanisms of qualitative platelet disorders?

Qualitative platelet disorders can occur as a result of impaired platelet adhesion, impaired platelet secretion, or impaired platelet aggregation.

566

Disorders of Platelet Adhesion

Which proteins are most important for platelet adhesion?

Platelet adhesion to subendothelial collagen is dependent on normal quantity and function of glycoprotein Ib (GPIb), VWF, and other specific platelet membrane collagen receptors such as glycoprotein VI (GPVI) (see Figure 28-2).2

What are the causes of impaired platelet adhesion?

The most common inherited bleeding disorder. Defective binding of VWF to platelets.

What is von Willebrand disease?

What peripheral blood smear finding is characteristic of Bernard-Soulier syndrome?

Von Willebrand disease (VWD).

Bernard-Soulier syndrome (BSS).

VWD is an inherited condition that occurs as a result of quantitative (types 1 and 3) or qualitative (type 2) defects of VWF, a protein necessary for platelet adhesion. The disease is relatively common in the general population with a prevalence of approximately 1%, but most cases are asymptomatic. VWD can also be acquired such as from lymphoproliferative disorders (eg, multiple myeloma) and cardiovascular conditions (eg, aortic stenosis). Hematomas, menorrhagia, and bleeding from minor trauma are the most common manifestations in adults. Other clues include excessive bleeding after surgery or dental procedures. Because VWF serves as a carrier protein for factor VIII, some patients with VWD have low levels of factor VIII activity, resulting in manifestations of hemophilia, including hemarthrosis and prolonged aPTT. Treatment for VWD may be necessary in the setting of clinical bleeding or prophylaxis for surgery. The goal is to normalize VWF and factor VIII levels exogenously with factor concentrate, or endogenously with desmopressin (1-deamino-8-D- arginine vasopressin [DDAVP]), which stimulates release of VWF from the Weibel-Palade bodies of endothelial cells.5,6 BSS is a rare autosomal recessive disorder that results in impaired platelet adhesion because of a congenital absence or dysfunction of GPIb. It typically presents early in life with bleeding manifestations that vary from mild to severe. Thrombocytopenia is variably present in patients with BSS. Giant platelets on peripheral blood smear is a hallmark feature (Figure 28-3). The diagnosis may be confirmed with platelet aggregation studies or flow cytometry.7

FIGURE 28-3 Giant platelet (arrow) in a patient with Bernard-Soulier syndrome. (From Pereira I, George TI, Arber DA. Atlas of Peripheral Blood: The Primary Diagnostic Tool. Philadelphia, PA: Wolters Kluwer Health; 2012.)

567

Disorders of Platelet Secretion

What is the role of platelet secretion in hemostasis?

Platelet adhesion stimulates platelet activation and secretion of substances, such as ADP and thromboxane A2, that promote platelet aggregation and the formation of a platelet plug (see Figure 28-2).

What are the causes of impaired platelet secretion?

Iatrogenic. Medication. A patient with chronic kidney disease Uremia. presents with new encephalopathy and has a pericardial friction rub. Look for electrocardiographic Osborn Hypothermia. waves (ie, J waves) (see Figure 19-3). A rare autosomal recessive condition Chédiak-Higashi syndrome.7 associated with oculocutaneous albinism, recurrent pyogenic infections, bleeding diathesis, neurologic disease (eg, ataxia), and large cytoplasmic granules in neutrophils and other granulocytes.

Which class of medication impairs platelet secretion?

What are the features of uremic bleeding diathesis?

What clinical settings can facilitate hypothermia-induced platelet dysfunction?

What are the storage pool diseases?

Nonsteroidal anti-inflammatory drugs (eg, aspirin), which are among the most commonly used medications in the world, impair platelet secretion by inhibiting cyclooxygenase enzymes that catalyze the generation of thromboxane A2 from arachidonic acid.2 Uremic patients may experience platelet dysfunction that results in benign manifestations such as ecchymoses, epistaxis, and bleeding gums, although serious complications such as overt Gastrointestinal bleeding, hemorrhagic pericarditis, and intracranial hemorrhage may occur. Mechanisms are multifactorial, and include impaired platelet secretion, adhesion, and aggregation. DDAVP can be useful in treating platelet dysfunction in uremic patients, particularly before surgical procedures.8 Hypothermia-induced platelet dysfunction may occur in the setting of metabolic disorders (eg, hypothyroidism, hypoglycemia, adrenal insufficiency), disturbed thermoregulation (eg, intracranial tumor), therapeutic interventions (eg, for cardiopulmonary bypass surgery or cardiac arrest from ventricular fibrillation), and environmental cold exposure. In addition to platelet dysfunction, hypothermia also causes platelet sequestration in the liver and spleen, resulting in thrombocytopenia. Platelet dysfunction and sequestration reverse on rewarming.9,10 The storage pool diseases are a heterogeneous group of conditions characterized by the abnormal presence or function of intracytoplasmic platelet granules involved in platelet secretion, usually resulting in mild to moderate bleeding diathesis. Conditions include gray platelet syndrome, Quebec platelet disorder, Hermansky-Pudlak syndrome, and Chédiak-Higashi syndrome.7

568

Disorders of Platelet Aggregation

Which proteins are most important for platelet aggregation?

Platelet aggregation is dependent on normal platelet adhesion, activation, and secretion, and the normal quantity and function of GPIIb-IIIa (ie, integrin αIIbβ3) and fibrinogen (see Figure 28-2).

What are the causes of impaired platelet aggregation?

A 48-year-old Antiplatelet medication (eg, clopidogrel). man with coronary artery disease presents with recurrent epistaxis after recent drug-eluting stent placement. Defective platelet- Fibrinogen disorders. platelet binding in the setting of normal GPIIb- IIIa. Defective platelet- Glanzmann’s thrombasthenia. platelet binding in the setting of normal fibrinogen.

What is the mechanism of clopidogrel-induced platelet dysfunction? What are the fibrinogen disorders?

What is the inheritance pattern of Glanzmann’s thrombasthenia?

ADP receptor inhibitors such as clopidogrel impair platelet function by blocking ADP-mediated platelet aggregation. Other medications that affect platelet aggregation include GPIIb/IIIa inhibitors (eg, abciximab).2

Fibrinogen disorders result from either quantitative abnormalities (eg, afibrinogenemia, hypofibrinogenemia) or qualitative abnormalities (eg, dysfibrinogenemia). These disorders may be inherited or acquired from conditions such as liver disease, disseminated intravascular coagulation (DIC), or malignancy (eg, renal cell carcinoma). Clinical manifestations may include bleeding, thrombosis, or both. Patients with fibrinogen disorders often have prolonged PT/INR and aPTT assays. More specific diagnostic studies include serum fibrinogen activity level and thrombin time. Additional confirmatory tests include thrombin activity-antigen ratio, thrombin time, 1:1 mixing study, fibrinogen electrophoresis, and fibrinogen gene analysis.11 Glanzmann’s thrombasthenia is a rare autosomal recessive disorder that occurs as a result of quantitative and/or qualitative abnormalities of the platelet glycoprotein GPIIb-IIIa. Manifestations of bleeding include purpura, epistaxis, gingival bleeding, and menorrhagia. The severity and frequency of bleeding events are variable. Prognosis is excellent for most adult patients. However, life-threatening bleeding can occur, particularly in association with trauma or other diseases (eg, cancer).12

569

Quantitative Platelet Disorders

What is a normal peripheral platelet count?

What is pseudothrombocytopenia?

What are the clinical manifestations of thrombocytopenia?

What are the 2 general mechanisms of thrombocytopenia?

What laboratory test may be helpful in distinguishing between impaired platelet production and Increased destruction? What are immature platelets?

Thrombocytopenia describes a platelet count in the lower 2.5th percentile of the normal platelet count distribution. Traditionally, the lower limit of normal platelet count has been defined as 150 K/µL. However, counts between 100 and 150 K/µL may be considered normal if stable for >6 months.13 Pseudothrombocytopenia is a laboratory artifact that occurs when the additive agent ethylenediaminetetraacetic acid (EDTA) triggers in vitro platelet clumping, generating a spuriously low platelet count as measured by the automated counter. Approximately 0.1% of the general population has EDTA-dependent antiplatelet antibodies that induce clumping. Review of the peripheral blood smear or use of a non-EDTA additive in these patients will provide an accurate platelet count.14 Patients with platelet counts >50 K/µL are generally asymptomatic; counts 30 to 50 K/µL may be associated with easy bruising; counts 10 to 30 K/µL may be associated with spontaneous bruising, spontaneous mucosal bleeding (eg, epistaxis), and prolonged bleeding after trauma; and counts <10 K/µL may be associated with spontaneous intracranial hemorrhage.15,16 Thrombocytopenia can occur as a result of Decreased platelet production or Increased platelet destruction.

The immature platelet fraction (ie, reticulated platelets) of peripheral blood may be helpful in determining whether thrombocytopenia is related to Decreased production (immature platelet fraction is low or normal) or Increased destruction (immature platelet fraction is elevated).

Immature platelets refer to platelets that have recently been released from the bone marrow. These platelets contain more ribonucleic acid (RNA) than mature platelets and can be measured by modern hematology analyzers. The immature platelet fraction reflects the megakaryopoietic activity of the bone marrow. In the setting of thrombocytopenia caused by peripheral destruction, healthy bone marrow responds by increasing megakaryopoiesis, which is reflected by an elevated immature platelet fraction. A fraction that is low or within normal limits suggests that the bone marrow is not responding appropriately (ie, impaired platelet production).17

570

Decreased Platelet Production

What are the causes of Decreased platelet production?

A 42-year-old intravenous drug user presents with fatigue, jaundice, spider angiomas on the chest, symmetric abdominal distension, lower extremity edema, elevated prothrombin time, and thrombocytopenia. A decrease in the proportion of bone marrow hematopoietic cells in the absence of an infiltrative process. A patient with a history of gastric bypass surgery develops peripheral neuropathy, pancytopenia, and hypersegmented neutrophils on peripheral blood smear. The peripheral blood smear demonstrates teardrop-shaped red blood cells, nucleated red blood cells, and immature white blood cells.

What is the mechanism of Decreased platelet production related to liver disease? What are the causes of bone marrow hypoplasia?

What are the causes of ineffective erythropoiesis?

What are the causes of bone marrow infiltration?

Liver disease caused by chronic hepatitis C virus infection.

Bone marrow hypoplasia.

Megaloblastic anemia.

Bone marrow infiltration.

The liver produces thrombopoietin, which stimulates the production and differentiation of megakaryocytes into platelets.2 Bone marrow hypoplasia can be transient or permanent, the latter of which is referred to as aplastic anemia. It is usually associated with pancytopenia. Causes of bone marrow hypoplasia include medications (eg, linezolid), toxins (eg, alcohol), infections (eg, parvovirus B19), and pregnancy. Most cases of aplastic anemia are idiopathic.18,19 Ineffective erythropoiesis describes the failure to produce mature blood cells as a result of dysfunctional progenitor cells, despite the presence of normo- or hypercellular bone marrow. It is often associated with pancytopenia. Causes include megaloblastic anemia and myelodysplasia.19 Bone marrow infiltration describes the replacement of hematopoietic stem cells. It is usually associated with pancytopenia. Causes include malignancy (eg, acute leukemia), infection (eg, tuberculosis), and myelofibrosis. Bone marrow infiltration leads to extramedullary hematopoiesis, which is associated with characteristic peripheral blood smear findings, including giant platelets (see Figure 27-5).19

571

Increased Platelet Destruction

What are the causes of Increased platelet destruction?

Increased activity at the normal site of platelet elimination from the body. A previously healthy 38-year-old man presents with bleeding gums and bruising after an upper respiratory infection and is found to have a platelet count of 8 K/µL. Hemolytic anemia, thrombocytopenia, and schistocytes on peripheral blood smear. Iatrogenic.

What is hypersplenism?

What is immune thrombocytopenic purpura?

What is the difference between thrombotic microangiopathy and microangiopathic hemolytic anemia (MAHA)? What are the clinical manifestations of heparin-induced thrombocytopenia?

Hypersplenism.

Immune thrombocytopenic purpura (ITP).

Thrombotic microangiopathy (TMA).

Heparin-induced thrombocytopenia (HIT).

Hypersplenism refers to the combination of splenomegaly, peripheral cytopenia(s), and normo- or hypercellular bone marrow. Massive spleens are capable of sequestering up to 90% of the peripheral platelet mass. Partial splenic embolization can be helpful in treating the thrombocytopenia of hypersplenism. The success of the procedure depends on the extent of embolization (embolization <50% is associated with relapse, while embolization >70% is associated with higher complication rates).2,19-21 ITP is an acquired disorder characterized by autoantibody-mediated platelet destruction that results in thrombocytopenia. It can be either primary, or secondary to a variety of conditions (eg, systemic lupus erythematosus). In adults, incidence follows a bimodal distribution with peaks in young adults and the elderly. Cases may be triggered by antecedent infection, particularly when viral. Clinical manifestations are variable, but can include severe and life-threatening thrombocytopenia. In adults, ITP is usually a chronic condition. Treatment is generally reserved for patients with platelet counts <30 K/µL or those with active bleeding. Glucocorticoids are first-line therapy. Other options include intravenous immune globulin (IVIG), anti-Rh immune globulin (in patients who are Rh-positive), rituximab, thrombopoietin receptor agonists, and splenectomy.22 MAHA refers to any cause of intravascular red blood cell fragmentation that results in hemolytic anemia, and generates schistocytes that can be identified on the peripheral blood smear. TMA refers to a group of conditions with characteristic endothelial and blood vessel wall abnormalities associated with arteriolar and capillary thrombosis, which results in MAHA, thrombocytopenia, and organ damage.23

HIT is characterized by a decrease in platelet count by more than half after exposure to heparin. It usually occurs 5 to 10 days after exposure, but in cases of reexposure (particularly within 30 days) it can occur sooner (rapid-onset HIT). It can also develop up to 3 weeks after heparin has been discontinued (delayed-onset HIT). Unlike most causes of thrombocytopenia in which bleeding dominates the clinical presentation, patients with HIT experience thrombotic complications related to a hypercoagulable state, such as deep vein thrombosis, pulmonary embolism, peripheral arterial thrombosis, and stroke. A high index of suspicion is required to recognize delayed-onset HIT because heparin exposure is relatively remote.24

572 Case Summary A previously healthy 39-year-old man presents with fever and skin rash, severe thrombocytopenia, renal dysfunction, and evidence confusion and is found to have a dependent petechial and purpuric of hemolytic anemia with an abnormal peripheral blood smear.

What is the most likely cause of thrombocytopenia in this patient? Thrombotic thrombocytopenic purpura (TTP).

573

Bonus Questions

present on the peripheral (see Figure 28-1, arrows). Schistocytes are a marker of traumatic intravascularfred blood cell fragmentation (ie, MAHA), occurring in conditions,such as TMA, What abnormality is The peripheral blood smear in this case demonstrates schistocytes, which are ragmented red blood cells that take the form of triangles, helmets and crescents blood smear in this mechanical lysis (eg, from a mechanical heart valve), and march hemoglobinuria. 25 case? What is thrombotic TTP is a form of TMA. Ittoccurs when there is functional deficiency of the metalloprotease knownras ADAMTS13 (a disintegrin and metalloproteinase with a What causes thrombotic TTP can be inherited or acquired, the latter of which is far more common and often associated with autoantibodies that inhibit ADAMTS13. One-half of thrombocytopenic thrombospondin type 1 motif, member 13). ADAMTS13 is responsible for cleaving large multime s of VWF. Uncleaved VWF leads to Increased platelet purpura? aggregation on the endo helial surface and microthrombi formation. ADAMTS13 activity <10% is specific for TTP. 26 thrombocytopenic acquired TTP cases are associated with an underlying systemic condition, including bacterial infection, autoimmune disease (eg, systemic lupus purpura? erythematosus), pregnancy, drugs (eg, clopidogrel), human immunodeficiency virus infection, pancreatitis, malignancy, and organ transplantation. One-half of cases remain idiopathic. 26 What clinical features in The clinical pentad of TTP consists of fever, thrombocytopenia, MAHA, neurologic manifestations, and renal insufficiency, most of which are present in this diagnosis of thrombotic with associated bleeding manifestations (eg, petechiae), MAHA with schistocytes onithe peripheral blood smear, and neurologic manifestations (eg, confusion, this case suggest the case. The complete pentad is present in <10% of patients with acute TTP. The most reliable clinical findings are severe thrombocytopenia (usually <30 K/µL) purpura? thrombocytopenic headache, stroke, seizure, coma). Other common manifestations include myocardial schemia and mesenteric ischemia. 26 What other conditions Other forms of TMA should be considered in this case. Primary TMA syndromes include TTP, HUS, and drug-induced TMA. Secondary TMA syndromes

should be considered in occur as a result of a variety of underlying conditions, including DIC, severe hypertension, HELLP syndrome (hemolysis, elevated liver enzymes, low platelet this case? count), systemic infection (eg, human immunodeficiency virus), systemic malignancy, autoimmune disorders (eg, systemic lupus erythematosus, antiphospholipid syndrome), severe vitamin B12 deficiency, and hematopoietic stem cell or organ transplantation.23 How should acquired Therapeutic plasma exchange to clear the blood of autoantibodies against ADAMTS13 is first-line therapy for acquired TTP. It should be performed daily until thrombotic there is improvement and stabilization of platelet counts, and resolution of hemolysis and end-organ dysfunction. Glucocorticoids and rituximab are often thrombocytopenic? initiated in parallel with plasma exchange.26 purpura be treated What is the prognosis of With prompt recognition and initiation of therapy, the average survival rate after an initial episode of TTP approaches 90%. However, a significantiproportion thrombotic of survivors suffer persistent morbidity such as neurocognitive deficits, arterial hypertension, and major depression. Almost one-half of patients w th acquired thrombocytopenic TTP experience at least 1 relapse. 26 purpura?

574 Key Points

Platelets are importanttfor maintaining the integrity of the vascular Clinical manifestations of platelet disorders include easy bruising, Bleeding time and modern platelet analyzers may be helpful in Platelet disorders can be qualitative or quantitative. Platelets are produced in the bone marrow and destroyed in the reticuloendothelial sys em after circulating for about 9 days. system and achieving primary hemostasis. excessive bleeding, and petechial or purpuric rash. identifying a platelet disorder. Qualitative platelet disorders can occur as a result of impaired platelet adhesion, impaired platelet secretion, or impaired platelet Thrombocytopenia can occur as a result of Decreased platelet aggregation. production or Increased platelet destruction. The immature platelet fraction may be helpful in determining whether thrombocytopenia is related to Decreased platelet production (immature platelet fraction is low or normal) or

Increased destruction (immature platelet fraction is elevated).

575

References 1. Leeksma CH, Cohen JA. Determination of the life of human blood platelets using labelled diisopropylfluorophosphanate. Nature. 1955;175(4456):552-553.

  1. Longo DL, Fauci AS, Kasper DL, Hauser SL, Jameson JL, Loscalzo J, eds. Harrison’s Principles of Internal Medicine. 18th ed. New York, NY: McGraw-Hill; 2012.

  2. Leung AK, Chan KW. Evaluating the child with purpura. Am Fam Physician. 2001;64(3):419-428. Francis J, Francis D, Larson L, Helms E, Garcia M. Can the Platelet Function Analyz

  3. (PFA)-100 test;substitute for the template bleeding time in routine clinical practice? er 5. Federici AB, Rand JH, Bucciarelli P, et al. Acquired von Willebrand syndrome: data Platelets. 1999 10(2–3):132-136. from an international registry. Thromb Haemost. 2000;84(2):345-349.

  4. Leebeek FW, Eikenboom JC. Von Willebrand’s disease. N Engl J Med. 2016;375(21):2067-2080. D’Andrea G, Chetta M, Margaglione M. Inherited p atelet disorders:

  5. thrombocytopenias and thrombocytopathies. BloodlTransfus. 2009;7(4):278-292.

  6. Boccardo P, Remuzzi G, Galbusera M. Platelet dysfunction in renal failure. Semin Thromb Hemost. 2004;30(5):579-589. Michelson AD, MacGregor H, Barnard MR, Kestin AS, Rohrer MJ, Valeri CR.

  7. Reversible inhibition of;human platelet activation by hypothermia in vivo and in vitro. 10. Van Poucke S, Stevens K, Marcus AE, Lance M. Hypothermia: effects on platelet Thromb Haemost. 1994 71(5):633-640. function and hemostasis. Thromb J. 2014;12(1):31.

  8. Cunningham MT,.Brandt JT, Laposata M, Olson JD. Laboratory diagnosis of dysfibrinogenemia Arch Pathol Lab Med. 2002;126(4):499-505.

  9. Nurden AT. Glanzmann thrombasthenia. Orphanet J Rare Dis. 2006;1:10.
  10. Stasi R. How to approach thrombocytopenia. Hematology Am Soc Hematol Educ Program. 2012;2012:191-197.

  11. Vicari A, Banfi G, Bonini PA. EDTA-dependent pseudothrombocytopaenia: a 12- month epidemiological study. Scand J Clin Lab Invest. 1988;48(6):537-542.

  12. McMillan R. Therapy for.adults with refractory chronic immune thrombocytopenic purpura. Ann Intern Med 1997;126(4):307-314.

  13. Thachil J, Fitzmaurice D. Thrombocytopenia in an adult. BMJ. 2013;346:f3407.
  14. Hoffmann JJ. Reticulated platelets: analytical aspects and clinical utility. Clin Chem Lab Med. 2014;52(8):1107-1117.

  15. Laboratory studies in drug-induced pancytopenia. Br Med J. 1980;280(6212):429-430. 19. Weinzierl EP, Arber DA. TheJdifferential diagnosis and bone marrow evaluation of 20hypersplenism treated with partial splenic embolisation. BMJ Case Rep. 2013;2013. to 21. Paz YMHL, Gonzalez-Estrada A, Alraies MC. Massive splenomegaly. BMJ Case Rep. new-onset pancytopenia. Am Clin Pathol. 2013;139(1):9-29. . Hanafiah M, Shahizon AM, Low SF, Shahrina MH. Severe thrombocytopenia due 2013;2013.

  16. Lambert MP, Gernsheimer TB. Clinical updates in adult immune thrombocytopenia. Blood. 2017;129(21):2829-2835.

  17. George JN, Nester CM. Syndromes of thrombotic microangiopathy. N Engl J Med.

576

2014;371(7):654-666. 24. Greinacher A. Clinical practice. Heparin-induced thrombocytopenia. N Engl J Med. 2015;373(3):252-261.

  1. Greer JP, Foerster J, Rodgers GM, et al, eds. Wintrobe’s Clinical Hematology. 12th ed. Philadelphia, PA: Lippincott Williams & Wilkins, A Wolters Kluwer Business; 2009.

  2. Joly BS, Coppo P, Veyradier A. Thrombotic thrombocytopenic purpura. Blood. 2017;129(21):2836-2846.